Jmet 2007 Original Paper

  • Uploaded by: Brian Williams
  • 0
  • 0
  • July 2020
  • PDF

This document was uploaded by user and they confirmed that they have the permission to share it. If you are author or own the copyright of this book, please report to us by using this DMCA report form. Report DMCA


Overview

Download & View Jmet 2007 Original Paper as PDF for free.

More details

  • Words: 21,984
  • Pages: 59
JMET 2007 Original Paper SECTION 1: VERBAL COMMUNICATION (1 – 40) Directions: Questions 1 to 4 relate to the following article: Number of words in this passage : 385

Pick up a glossy magazine or newspaper supplement and there will almost certainly be at least one double page spread that looks like a regular editorial page but is headed up either 'promotion' or 'advertisement'. These hybrids – unattractively but aptly called advertorials – are being used with increasing frequency by a growing number of companies. Traditionally the preserve of hightechnology clients with a complicated message to get across to potential customers, the use of this technique has now spread to sectors like financial services, alcohol and automobiles. One major reason why marketing departments are becoming more receptive to ideas for advertorials is that publishers are pursuing them more aggressively at a time of shrinking ad budgets, while they are being treated far more professionally in a bid to persuade clients that this is a creative opportunity to spread their message to their target audiences. Pouring more imagination into them allied with raising production standards has also been a means whereby the commercial executives of magazines and newspapers can try to convince skeptical editors who strongly disapprove of blurring the advertising / editorial line of their worth. What advertorials are about is control – controlling the message in an editorial format. Positive editorial coverage of a company and / or its products in credible publications is the best publicity any company can hope for, but often proves elusive. A successful advertorial can pinpoint the way the company delivers its message to the heart of its target audience. High technology was one of the main sources of early advertorials – unsurprisingly, the products are complex and need to be explained with some technical detail to get the story across. That is not so easy with traditional advertising. Advertorials can also to some degree circumvent journalistic indifference to what a company is doing because editorial coverage has already been so extensive. For example, in the case of a company like Compaq, whose swift growth in the computer market attracted many inches of editorial space, that very success can lead to journalists wondering how they can write something different about Compaq. There can be diminishing returns from an editorial point of view. So advertorials let the company present things editorially but with bought space. While they should be strongly labeled, information is being given to readers in a format that looks familiar. 1. In the light of your reading of the passage above, identify the option that contains the set of words CLOSEST in meaning to the set of words inCAPITALS: SKEPTICAL : CIRCUMVENT : ELUSIVE (A) incredulous : surround : baffling (B) doubtful : avoid : evasive (C) thoughtful : deceit : illustrative (D) philosophical : revolve : deceptive Explanatory Note: ‘Sceptical’ – as used in the last sentence of para 2 – means ‘doubtful’. ‘Circumvent’ – as used in the first line of the last para – means ‘avoid’ (it cannot mean ‘surround’, ‘deceit’ or ‘revolve’). ‘Elusive’ – as used in line 3 of para 3 – means ‘evasive’. Choice (B) 2. In the above passage, the phrase "blurring the advertising / editorial line or their worth" implies (A) diluting the perceived quality of their editorials. (B) hiding the actual value of the paper. (C) obscuring the actual facts in the paper. (D) devaluing the advertising potential of the editorials.

Explanatory Note: Refer to the last sentence of para 2 – editors disapprove of advertorials – blurring the line between advertising and editorial – because the perceived quality of the actual editorial gets diluted. Choice (A) 3. The passage DOES NOT discuss (A) attitude of journalists towards advertising. (B) advertorials and the publishing industry. (C) use of advertorials in industries. (D) impact of new technologies on advertorials. Explanatory Note: Option C is not discussed in the passage. Option C says ‘use of advertorials in industries’ – whereas the passage talks of advertorials in magazines or newspapers by industries. Choice (C) 4. According to the passage, (A) high technology does not support traditional advertising. (B) traditional journalists are indifferent to advertorials. (C) advertorials facilitate advertising of complex products in a professional manner. (D) advertorials occupy double-page spreads in magazines. Explanatory Note: Option D is para.

clearly

stated

in

the

first

sentence

of

the first Choice (D)

Directions: In Question 5 and 6, choose from the respective options the CORRECT PASSIVE form of the active sentence: 5. She resembles a Greek goddess. (A) A Greek goddess resembled her (B) A Greek goddess resembles her (C) She is resembled by a Greek goddess (D) None of the above Solution: The sentence does not have an object it has only a compliment for the verb. Hence it cannot be rendered in the passive voice. Choice (D) 6. Mr.Sullivan made a major error in the estimation. (A) A major error was made in the estimate by Mr.Sullivan (B) A major estimate was made in the error by Mr.Sullivan (C) A major error was being made in the estimate by Mr.Sullivan (D) A major error was estimated by Mr.Sullivan. Solution: When we change from active to passive, the object becomes the subject. Here the object is ‘an error’ (made what? Made an error). Only option A is right. Option B is semantically incorrect, option C has tense error and option D is again semantically incorrect. Choice (A) Directions: In Questions 7 and 8, select the set of words from the given options that BEST expresses a relationship SIMILAR to the set in CAPITALS: 7. LIONS : PRIDE : FOREST (A) geese : gaggle : sky (B) houses : colony : city (C) fish : shoal : ocean (D) paper : ream : press Solution: ‘Pride’ is the collective noun for ‘lions’ and ‘lion’ are in forest. Similarly ‘shoal’ is the collective noun for ‘fish’ and fish are in ocean. Hence option C. Option B is not the answer because ‘colony’ can

be a city.

collection

of

‘houses’

but

a

house can be Choice (C)

any

where



not

only

in

8. POET : VISION : PROPHET (A) politician : constituency : voter (B) killer : violence : terrorist (C) student : school : principal (D) plant : herbivore : food-chain Solution: A ‘poet’ has ‘vision’ – he can be a ‘prophet’. Similarly a ‘killer’ has ‘violence’ and he can be a ‘terrorist’. Choice (B) Directions: In Question 9, choose the option which is CLOSEST in meaning to the sentence given below: He was convinced that it would take at least fifty years before a few men would understand what he had accomplished; and he feared that even then his teachings would be misinterpreted and misapplied. 9. (A) He was convinced that his teachings would be misinterpreted and misapplied after fifty years. (B) He was convinced that after misinterpreting and misapplying his teachings for fifty years, they would be appreciated. (C) He was convinced that understanding his teachings would lead to their misinterpretation and misapplication. (D) He was convinced that understanding and applying his teachings would take at least fifty years.

Solution: The given sentence says it would take at least fifty years for him to be understood – though even after that he would be misinterpreted and misapplied. Option A changes the idea with ‘after’ and does not talk of being understood. Option B says he would be appreciated after fifty years. Option C is also off the mark – understanding does not lead to misinterpretation and misapplication. Only option D is correct – the main idea is right though the subsidiary idea of misinterpretation and misapplication is left out. Choice (D)

Directions: Read the passage carefully and answer the Questions 10 to 12 that follow.

Number of words in this passage : 551

I feel that this award was not made to me as a man, but to my work – a life's work in the agony and sweat of the human spirit, not for glory and least of all for profit, but to create out of the materials of the human spirit something which did not exist before. So this award is only mine in trust. It will not be

difficult to find a dedication for the money part of it commensurate with the purpose and significance of its origin. But I would like to do the same with the acclaim too, by using this moment as a pinnacle from which I might be listened to by the young men and women already dedicated to the same anguish and travail, among whom is already that one who will some day stand here where I am standing.

Our tragedy today is a general and universal physical fear so long sustained by now that we can even bear it. There are no longer problems of the spirit. There is only the question: When will I be blown up? Because of this, the young man or woman writing today has forgotten the problems of the human heart in conflict with itself which alone can make good writing because only that is worth writing about, worth the agony and the sweat.

He must learn them again. He must teach himself that the basest of all things is to be afraid; and, teaching himself that, forget it forever, leaving no room in his workshop for anything but the old verities and truths of the heart, the old universal truths lacking which any story is ephemeral and doomed – love and honor and pity and pride and compassion and sacrifice. Until he does so, he labors under a curse. He writes not of love but of lust, of defeats in which nobody loses anything of value, of victories without hope and, worst of all, without pity or compassion. His griefs grieve on no universal bones, leaving no scars. He writes not of the heart but of the glands.

Until he relearns these things, he will write as though he stood among and watched the end of man. I decline to accept the end of man. It is easy enough to say that man is immortal simply because he will endure: that when the last ding-dong of doom has clanged and faded from the last worthless rock hanging tideless in the last red dying evening, that even then there will still be one more sound: that of his puny inexhaustible voice, still talking. I refuse to accept this. I believe that man will not merely endure: he will prevail. He is immortal, not because he alone among creatures has an inexhaustible voice, but because he has a soul, a spirit capable of compassion and sacrifice and endurance. The poet's, the writer's duty is to write about these things. It is his privilege to help man endure by lifting his spirit, by reminding him of the courage and honor and hope and pride and compassion and pity and sacrifice which have been the glory of his past. The poet's voice need not merely be the record of man, it can be one of the props, the pillars to help him endure and prevail. 10. The phrase "labours under a curse" in paragraph 3 means that the young writer (A) is under a curse, so to speak. (B) continues to work though he is cursed. (C) is condemned to be abject. (D) is given to lusts.

Explanatory Note: The phrase ‘labour under something’ means to believe something that is not true. Hence ‘labour under a curse’ means ‘to be under a curse, so to speak’ – not really a curse but ‘as

if’. Choice (A)

11. Which of the following inferences CANNOT be drawn from the passage? (A) Good writing is always about the conflicted human heart. (B) A writer should overcome his fear and advocate the universal truths. (C) A writer should not seek money or fame. (D) A writer should espouse the immortality of the human soul.

Explanatory Note: Option C – A writer should not seek money or fame cannot be inferred from the passage. Although the author does not seek glory or profit he does not say or imply that other writers should not do so. The others are stated or implied. Choice (C)

12. Which of the following is the MOST APT title for this passage? (A) The Tragedy of Mankind (B) Human Heart in Conflict (C) The Writer's Duty (D) The Spirit of Man

Explanatory Note: The passage passage).

in

about

the

writers

duty

(see

the

5th line

from the end Choice (C)

of

the

Directions for Questions 13 to 15: Identify the grammatically CORRECT AND APPROPRIATE option. 13. (A) As the boys approached the swamp, frogs could be heard croaking. (B) She put her car in the garage because she never leaves it out when it is bad weather. (C) In early colonial villages, you had to depend on wood for fuel. (D) Many students who major in mathematics today find employment with computer companies. Solution: Option A is incorrect because it is partly in active voice and party in passive voice. Option C is wrong – it should be ‘In villages in early colonial era………’. In option D, ‘today’ is wrongly positioned – it can begin the sentence or be used after ‘Many students’. Option B is right. Choice (B) 14. (A) Because we put a wire fence around the chicken yard, the chickens cannot escape. (B) The disadvantages of credit cards can offset the advantages, which merits careful consideration. (C) When Madhuri visited her mother, she had a cold

(D) These sort of things happen. Solution: Option A is incorrect – it should be ‘Because we have put …..’. Option D is incorrect – it should be ‘These sorts of things …..’ not singular ‘sort’ with plural ‘these’. Option B is incorrect – the relative pronoun ‘which’ is inappropriate since there is no subject for it. It should be ‘and this’ instead of ‘which’. Option C is grammatically correct – although there is ambiguity in who had cold, Madhuri or her mother. Choice (C) 15. (A) The school is opposite to the township building. (B) In chapter 1, she accepts her first job as a kitchen maid, but by chapter 3, she is cooking for an Indian prince. (C) Hold the rifle firmly against your shoulder, and then you should take careful aim. (D) I like an occasional cup of coffee, for they give me an added lift. Solution: Option A is incorrect – it should be ‘opposite’ not ‘opposite to’. Option C is incorrect – the sentence begins as an instruction (Hold…..) and so ‘you should’ in the second part is inappropriate and must be dropped from the sentence. Option D has two errors – the plural ‘they’ is incorrect with the singular ‘cup of coffee’ and ‘lift’ is inappropriate (may be vigour would be better). Only option B is correct. Choice (B) Direction: In Question 16, identify the option with CORRECT punctuation. 16. (A) The people of this company, have, always been aware of the need, for product's of better quality and lower prices. (B) The new residents of Canada faced still more hardships; loneliness, life in a wilderness, even death. (C) In April 1789 the ship left Ceylon with its cargo. (D) A fairy story as distinct, from a merry tale or an animal story, is a serious tale with a human hero and happy ending. Solution: Sentence A is incorrect – all the three commas make the punctuation wrong. The commas are not needed. In option C, there should be a comma after 1789. In Option D, the comma should be after ‘story’ not after ‘distinct’ Option B is right – the semicolon (or a dash) is used to separate what illustrates. Choice (B) Directions: Question 17 and 18 relate to the passage given below. Number of words in this passage : 512

With each passing day, it is getting easier to believe that the acceleration in India's economic growth from around 6% to 8% is here to stay. The hard part is trying to explain why this has happened. How this is explained is important since it has a bearing on our future policy. As per conventional wisdom, India's growth accelerated to around 6% in the nineties from the historical rate of 3.5% because 'reforms' had unleashed the pent-up energies of Indian entrepreneurs long shackled by the socialist raj. It slowed subsequently because 'reforms' had lost momentum. The last three years' spurt in growth is the fortuitous result of a global economic boom. Once the world economy slows down, we will be back to 6% growth unless we proceed with 'second generation' reforms. However, each of these propositions bristles with problems. It is not true that economic growth rate accelerated to 6% in the nineties. In fact, research has shown that the 'structural break' in India's economic growth occurred not in the early nineties but in the eighties, when economic growth accelerated to close to 6%. The growth in the first decade after reforms was not significantly different from the growth rate in the eighties. The 'reforms' in the sense of market-oriented or even pro-

business policies did not commence overnight in 1991, but had commenced earlier. Economic policies in the nineties merely helped consolidate an underlying trend. Subsequently, the world economy slowed down in 2001-03, which put the brakes on Indian economy. Then came the crucial change, an acceleration to 8% in 2004-06. This cannot be ascribed to any fresh bout of 'reforms' or even to the global boom. There have been important structural changes in the economy. One is the rise in the savings rate from 23.5% in 2001-01 to 29.1% in 2004-05. Most of this increase has come from the turnaround in public savings. Thanks to the rise in the savings rate, the economy has moved on to an altogether higher investment rate. The second structural change is enhanced export competitiveness, reflected in the rising share of exports. The total exports (trade plus invisible receipts) / GDP ratio has risen sharply from 16.9% in 2000-01 to 24.6% in 2005-06. A third, less noticed change in recent years is financial deepening. The bank assets / GDP ratio rose from 48% in 2000-01 to 80% in 2005-06 on the back of a surge in bank credit. One factor is common to these three structural changes: lower interest rates. The decline in interest rates has helped fiscal consolidation, it has boosted firms' competitiveness and it has led to a huge increase in retail credit. Lower interest rates have been made possible by the rise in inflows on both current and capital accounts. The rise in inflows, in turn, reflects growing overseas confidence in India's economic potential – confidence created by two decades of economic growth of 6%. The sharp depreciation in the rupee in the nineties undoubtedly helped but it is worth recalling that a trend towards rupee depreciation was under way in the eighties itself. 17. Which of the following statement is INCORRECT according to the passage? (A) Growth rate after reforms was similar to that in the eighties. (B) Reforms in economic policies had started prior to the nineties. (C) Structural changes in the Indian economy have helped lower interest rates. (D) Increase in public savings rate has contributed to higher investment rates. Explanatory Note: Option B is incorrect, the others are all stated or can be inferred from the passage. Para 3 says the growth noticed in the 90s had started in the eighties. It also says pro-business policy ‘did not commence overnight in 1991, but had commenced earlier’. There is nothing to show the ‘earlier’ refers to the eighties – it may merely mean 1990. Choice (B) 18. The passage DOES NOT discuss (A) factors contributing to lower interest rates. (B) the importance of world economy on India’s reform rates. (C) dimensions of structural changes in India’s economic reforms. (D) the role of the public sector in India’s reforms. Explanatory Note: The passage all. (D)

does

not

mention

public

sectors

at Choice

Directions: Question 19 consists of four groups of jumbled phrases, of which only ONE is grammatically correct. Identify the CORRECT option. 19. (A) those command of the language are poor / other thing being equal / themselves effectively are sure to succeed / more rapidly than / people in any fields who can express (B) for whatever effects they may create / in analyzing prose rhythms or sentence movement / length, and interrelations of rhythmical units / it is always necessary to take into account not only the number, / but also the patterns of stressed and unstressed words (C) credence to our own recollections of events / accompanied by vivid compelling details / readily spring to mind and is / than other's when our memories / we are likely to give more (D) What time is left for living / than to see peoples working from morn till night / in cafes and small-talk / and then proceed to fritter away at card-tables / certainly is common nowdays. Solution:

Only option B makes a grammatically correct sentence – In analyzing prose rhythms or sentence movement, it is always necessary to take into accent not only the number, length, and interrelations of rhythmical units but also the patterns of stressed and unstressed words for whatever effects they may create. Option A must have ‘whose’ – ‘Those whose command of the language …..’. Options C and D do not make a grammatical sentences. Choice (B) Directions: In Questions 20 to 22, choose the option which is OPPOSITE in meaning to the CAPITALIZED word. 20. ANATHEMA (A) Salubrious se Solution: Anathema (blessing). (C)

(B) Feral

means

a

21. FECUND (A) Barren id

curse.

The

(B) Fertile

Solution: ‘Fecund’ ‘barren’. Choice (A)

means

22. CONCINNITY (A) Congruity Solution: Concinnity ‘mismatch’.

(C) Benediction

fertile

harmonious.

opposite

is

benediction Choice

(C) Auspicious

-

(B) Mismatch means

(D) Cur

The

(D) Stol

the

opposite

(C) Deceit word

that

is

(D) Harmony can

be

its opposite Choice (B)

is

Directions: In Question 23, identify the grammatically INCORRECT option. 23. (A) The meeting has been preponed by a week. (B) Either you or Ram is going to look after it. (C) The argument explains neither what went wrong nor how it should be put right. (D) Customers want not only good service but also courtesy. Solution: Option A is incorrect – the word ‘preponred’ is not used by native speakers – though it is quite commonly used in India. Choice (A) Directions: Question Nos. 24 to 26 relate to the passage given below. Number of words in this passage : 643

As a memory researcher, I have long been intrigued by the phenomenon of memory failures. What are the different ways that memory can get us into trouble? Bringing together everything I knew of memory's imperfections, lapses, mistakes and distortions, I hit on a way of thinking that helped to make things fall in place. I propose that memory's malfunctions can be divided into seven fundamental transgressions or "sins", which I call transience, absent-mindedness, blocking, misattribution, suggestibility, bias, and persistence. Just like the ancient seven deadly sins, the memory sins occur frequently in everyday life and can have serious consequences for all of us. Transience, absent-mindedness and blocking are sins of omission: We fail to bring to mind a desired fact, event or idea. Transience refers to a weakening or loss of memory over time. It is probably not difficult for you to remember now what you have been doing for the past several hours. But if I ask you

about the same activities six weeks, six months, or six years from now, chances are you will remember less and less. Transience is a basic feature of memory, and the culprit in many memory problems. Absent-mindedness involves a breakdown at the interface between attention and memory. Absentminded memory errors – misplacing keys or eye-glasses, or forgetting a lunch appointment – typically occur because we are preoccupied with distracting issues or concerns, and do not focus attention on what we need to remember. The desired information is not lost over time; it is either never registered in memory to begin with, or not sought after at the moment it is needed, because attention is focused elsewhere. The third sin, blocking, entails a thwarted search for information we may be desperately trying to retrieve. We have all failed to produce a name to accompany a familiar face. This frustrating experience happens even though we are attending carefully to the task at hand, and even though the desired name has not faded from our minds – as we become acutely aware when we unexpectedly retrieve the blocked name hours or days later. In contrast to these three sins of omission, the next four sins of misattribution, suggestibility, bias, and persistence are all sins of commission: some form of memory is present, but it is either incorrect or unwanted. The sin of misattribution involves assigning a memory to the wrong source: mistaking fantasy for reality, or incorrectly remembering that a friend told you a bit of trivia that you actually read about in a newspaper. Misattribution is far more common than people realize, and has potentially profound implications in legal settings. The related sin of suggestibility refers to memories that are implanted as a result of leading questions, comments, or suggestions when a person is trying to call up a past experience. Like misattribution, suggestibility is especially relevant to – and can sometimes create havoc – within the legal system. The sin of bias reflects the powerful influences of our current knowledge and beliefs on how we remember our pasts. We often unknowingly or unconsciously edit or rewrite our previous experiences in light of what we now know or believe. The result can be a skewed rendering of a specific incident, or even an extended period of our lives, which says more about how we feel now than about what happened then. The seventh sin – persistence – entails repeated recall of disturbing information or events that we would prefer to banish from our minds altogether: remembering what we cannot forget, even though we wish that we could. Everyone is familiar with persistence to some degree: recall the last time that you suddenly awoke at 3:00 AM, unable to keep out of your mind a painful blunder on the job or a disappointing result on an important exam. In more extreme cases of serious depression or traumatic experience, persistence can be disabling and even life-threatening. 24. The above passage DOES NOT mention (A) impact of memory malfunctions on daily lives. (B) reasons for memory malfunctions. (C) relationship between seven memory sins and seven deadly sins. (D) lapses and distortions of memory. Explanatory Note: The passage does not mention relationship between the seven dradly sins and memory sins. The comparison at the end of para 1 only says that like the deadly sins, memory sins occur. Choice (C) 25. The above passage implies that (A) sins of commission are more serious memory malfunctions than sins of omission. (B) the sin of bias arises as a result of misattribution. (C) the sin of persistence most frequently occurs when we are asleep. (D) sins of omission involve presence of memory in some form or other. Explanatory Note: The last three paras show that sins of commission are far more serious than sins of omission (words like ‘has potentially profound implication’, ‘can be disabling or even life-threatening’) show

that serious. Choice (A)

it

is

far

26. In the passage, the term "transience" refers to (A) transference. (B) truculence. (C) ephemeral. Explanatory Note: ‘Transience’ means ‘ephemeral’. e (C)

what

is

not

more

(D) epiphanic.

permanent

or

what

is Choic

Directions: In Questions 27 and 28, select the pair of words from the given options that best expresses a relationship SIMILAR to the pair inCAPITAL letters: 27. OBJURGATE : OBSECRATE (A) abdicate : abrogate (B) renegade : relegate (C) chide : supplicate (D) obfuscate : obligate Solution: ‘Objurgate’ means a harsh rebuke. ‘Obsecrate’ means to entreat or ‘supplicate’. The words are antonyms. So also ‘chide’ and ‘supplicate’. Choice (C) 28. RESTLESS : RESTIVE (A) flammable : inflammable (B) imminent : eminent (C) haunted : hunted (D) oculist : occultist Solution: ‘Restless’ and ‘inflammable’.

‘restive’

are

synonyms



so

also

‘flammable’ and Choice (A)

Directions: In Questions 29 and 30, fill in the blanks with the option that has the MOST APPROPRIATE set of words. 29. Children whose _____ survives parental discipline and who manage to grow up before they blow up are invited to _____ the university faculty. (A) inquisitiveness, visit (B) interest, address (C) curiosity, join (D) inquiry, join Solution: All the four choices could fit in the first blank. In the second blank only ‘visit’ seems possible. You wouldn’t ask children (even if they are grown up) to join or address the faculty. They can however, ‘visit’ the faculty’. Inquisitiveness’ is better than the other words for the first blank especially as the sentence says’ grow up before they blow up’ (suggesting the danger to which it exposes one). Choice (A) 30. For years, nuclear-power advocates have claimed that nuclear power is the most _____ form of energy available; but in light of a few facts, one begins to _____ this claim. (A) cheap, support (B) useful, question (C) expensive, contest (D) economical, doubt Solution:

The use of the word ‘claim’ suggests that there is room for ‘doubt’. Hence choice (D) Further ‘economical’ is apt in the first blank. The use of ‘most’ rules out ‘cheap’ (you would say cheapestnot most cheap), ‘expensive’ is the apposite of what is expected and ‘useful’ is not in doubt. Choice (D) Directions: Questions 31 to 33 relate to the passage given below. Number of words in this passage : 385

Dear Friend, your letter gently but unmistakably intimates that I am a slacker, a slacker in peace as well as in war; that when the World War was raging bitterly I dawdled my time with subjects like symbolic logic, and that now when the issues of reconstructing a bleeding world demand the efforts of all who care for the future of the human race, I am shirking my responsibility and wasting my time with Plato and Cicero. Your sweetly veiled charge is true, but I do not feel ashamed of it. On the contrary, when I look upon my professional colleagues who enlisted their philosophies in the war, who added their shrill voices to the roar of the cannons and their little drops of venom to the torrents of national hatreds, I feel that it is they who should write apologies for their course. For philosophers, I take it, are ordained as priests to keep alive the sacred fires on the altar of impartial truth, and I have but faithfully endeavored to keep my oath of office as well as the circumstances would permit. It is doubtless the height of the unheroic to worship truth in the bombproof shelter of harmless mathematics when men are giving their lives for democracy and for public order which is the basis of civilization. But it would be sad if all the priests deserted their altars and became soldiers, if the Sermon on the Mount were utterly erased to give place to manuals of bayonet practice or instructions on the use of poison gas. What avails it to beat the enemy if the sacred fires which we are sworn to defend meanwhile languish and die for want of attendance? 31. Which of the following is the MOST APPROPRIATE title for the passage? (A) Philosophy in wartime: An Apologia (B) Philosophy versus War (C) In defence of Philosophy (D) Phylosophy's quarrels with War Explanatory Note: The author in the passage defends philosophy and the right of philosophers to work on their subject even during wartime. Hence ‘In defence of philosophy’ is the appropriate title. Choice A is not suitable because the author feels that it is his colleagues – those who ‘added their shrill voices to the roar of the canon’ – who should write apologies. Choices B and D are inappropriate because the author is not looking at it as an ‘either …..or’ situation – either philosophy or war. Choice (C) 32. According to the passage, a philosopher should (A) always shun action and privilege speculation. (B) at all times promote the disinterested inquiry of his discourse. (C) stay away from ideologues. (D) support anti-war activism. Explanatory Note: In the passage, the author defends himself for not participating in the war and feels that the philosopher should ensure that the sacred fire does not die. The last four lines of the passage make choice A the right answer. (The word ‘privilege’, as a verb here, means favour). Note line 9 ‘For philosophers … are ordained as priests to keep alive the sacred fire………’ Choice (A) 33. Which of the following statements CANNOT be directly inferred from the passage? (A) The writer has disagreements with his professional colleagues. (B) The writer is aware of the sacrifices made in a war. (C) The writer considers philosophy a sacred calling. (D) The writer is a pacifist.

Explanatory Note: Choices A B and C can be inferred from the passage. But we cannot infer that the writer is a ‘pacifist’ (one is opposed to war or violence). While he does not want to enter the war, he sees that some people have to ‘……men are giving their lives for democracy and for public order which is the basis of civilization’. Hence we cannot infer that he is a pacifist. Choice (D) Directions for Question 34: Sentences in the following passage have been variously combined in the options given below. Choose the MOSTAPPROPRIATE AND CONCISE option. Keepers of private notebooks are a different breed altogether. They are lonely and resistant rearrangers of things. They are anxious malcontents. They are children afflicted at birth with some presentiment of loss. 34. (A) Keepers of private notebooks are a different breed altogether, lonely rearrangers of things, anxious malcontents, children afflicted at birth with some presentiment of loss. (B) Keepers of private notebooks, who are lonely, resistant rearrangers and anxious malcontents, are children afflicted at birth with some presentiment of loss. (C) Keepers of private notebooks, a different breed in being lonely, resistant rearrangers of things and anxious malcontents, are children afflicted at birth with some presentiment of loss. (D) None of the above Solution: Option B is concise (more so than A and C) and covers all the points in the given sentence. Choice (B) Directions: In Question 35, choose the option which DOES NOT have a similar meaning to the sentence given below: Some of the students reflect a growing confidence in their ability to manage successfully the demands placed upon them by their own ambitions, by their ability to construct intelligent messages, and by their listeners' often irascible modes of responsing. 35. (A) Listeners' response modes and their own personal ambitions place great demands on students. (B) Students construct intelligent messages to cope with listeners' demands. (C) Students are becoming increasingly capable of managing multiple demands placed on them. (D) Ability to construct intelligent messages helps students cope with increasing demands arising from personal ambitions and listeners' responses. Solution: The focus of the sentence is on the student’s ‘ability to manage successfully’. Only option A does not mention this and hence does not have a similar meaning as the given sentence. Choice (A) Directions: In Questions 36 to 38, choose the option that is CLOSEST in meaning to the CAPITALIZED word. 36. MIRIFIC (A) Marvellous Solution: ‘Mirific’ ‘marvellous’. 7. PLEBEIAN (A) Vulgar

(B) Mundane

(C) Mystical

(D) Morbid means

Choice (A) (B) Aristocratic

(C) Prophetic

(D) Certainty

Solution: ‘Plebeian’ ‘vulgar’. 38. FIDUCIAL (A) Official Solution: ‘Fiducial’ worthy’.

means

common

or

Choice (A) (B) Deceit

(C) Trustworthy means

(D) Parochial being

‘trust

Choice (C)

Directions: In Question 39, identify the option with INCORRECT spellings. 39. (A) On the twelfth of every month, the psychiatrists visit the organization. (B) Entering the sanctum of South Indian temples dressed in western clothes is considered sacreligious. (C) We try to accommodate as many students as possible in our hostels. (D) We received the mattress after repeated requests. Solution: Option B interchanged. (B)

has

‘sacrilegious’

incorrectly

spelt



‘e’

and ‘i' Choice

Directions: The direct speech in Question 40 is rewritten as reported speech (indirect form) in the options below. Identify the grammaticallyCORRECT option. 40. The President said to the General, "Is your army well supplied? Is it ready for battle?" (A) The President asked the General whether his army was well supplied and whether it was ready for battle (B) The President asked the General whether his army was well supplied and is it ready for battle (C) The President asked the General if his army is well supplied and if it is ready for battle (D) The President asked the General whether his army is well supplied and was it ready for battle Solution: Only option A is grammatically correct - the reported speech is in past tense if the reporting verb is in past tense (asked). Option B C and D all have the present tense with the use of ‘is’. Choice (A)

SECTION 2: LOGICAL REASONING (41 – 80) 41. The mushrooming of business schools in the country is a cause for shortage of faculty with Ph.D qualification. In addition, the higher pay and generous fringe benefits given by industry has encouraged qualified people to not seek academic positions. Which of the following statements, if true, would tend to STRENGTHEN the argument? (A) The average salary for industry positions in Gujarat is more than the average salary for faculty positions in some business schools in Ahmedabad by around 30%. (B) The average salary for industry positions in Gujarat is less than the average salary for faculty positions in a top business school in Ahmedabad by around 30%. (C) The average salary for recent Ph.D graduates in the industry is 20% higher than that in academics. (D) The rate of growth of salaries for the industry positions has been higher than the rate of growth of salaries for academic positions for the past three years. 42. You have five balls that look alike. Four of them have the same weight and are lighter than the fifth ball. What is the minimum number of times you need to weigh to identify the heavier ball with certainty? (A) 1 (B) 2 (C) 3 (D) 4 43. A pair of grouping of symbols is given below. Choose a pair of symbols that best expresses the relationship closest to the original pair. LLI:UQR

(A) DEF:ABC

(B) AXE:TIX

(C) AEF:BGO

(D) LMN:AEF

44. Charu is a person of regular habits. One day Charu saw that her clock had stopped working. She changed the battery and set it to some arbitrary time. Immediately after that, she walked to her friend Paru's house. On entering Paru’s house, she glanced at the clock that showed the correct time. Charu also noticed the time when she left Paru's place. As usual, she walked back to her house and reset her clock. If the time set by Charu is correct, which of the following assumptions have to be made? (A) Charu knows the exact distance to Paru's house. (B) Charu passed by a clock store on the way back to her house. (C) She walked at the same speed while going to and returning from Paru's house. (D) Charu knows the average time it takes to walk to Paru's house. Directions: Questions 45 and 46 are based on the following passage: Gopal: My father insists that the only way to get a good rank in the JMET examination is to work much harder than what I do at present. However, Alok and Raju, my two college seniors whom he coached for JMET last year got good ranks with less effort than what I am putting in. 45. Gopal's primary purpose for making his point is to (A) present the ideal method to prepare for JMET examination. (B) present evidence that was previously overlooked. (C) point out a logical flaw in his father's reasoning. (D) draw an analogy to justify his method of preparation. 46. Which of the following statements would be the most effective rebuttal by Gopal's father to his arguments? (A) Your two college seniors did not have to put in more effort as they had been preparing for this examination for a longer duration. (B) I have been coaching students for this examination since its inception, and hence I feel that you need to put in more effort. (C) You need to provide much more detailed data to support your argument. (D) My suggestion is not obviously wrong. There is only one way to find out if it is wrong, and that is to try it. Directions: Questions 47 and 48 are based on the following passage: It is important for companies to motivate their employees to say fit. Therefore, employees should be provided with gymnasium facilities. 47. Which of the following, if true, most strengthens the above argument? (A) Only those employees who use gymnasium facilities are fit. (B) The employees who use gymnasium facilities are fit. (C) The employees who are fit use gymnasium facilities. (D) Some employees who use the gymnasium facilities are fit. 48. Which of the following, if true, most weakens the above argument? (A) All those employees who undergo a dieting program are fit. (B) Not all those employees who undergo a dieting program are fit. (C) Not all those employees who use gymnasium facilities are fit. (D) All of those employees who use gymnasium facilities are fit. 49. Sanjay: I just heard that Sachin got out for zero (0) runs in the first innings of the second test against Bangladesh. Rajeev: That can't be true. He had scored two centuries in the last two innings that he had played. From the conversation above it can be inferred that (A) Rajeev thinks that Sanjay is lying. (B) Rajeev thinks that no one who had scored two centuries in the last two innings could possibly get out for zero runs in the next innings. (C) Rajeev concludes that Sachin is inconsistent since he got out for zero after scoring two hundreds in the last two innings. (D) Sanjay knows Sachin got out for zero runs.

Directions for Questions 50 to 53. These questions are presented with three true statements: Fact 1, Fact 2, and Fact 3. Then, you are given three more statements (labeled I, II and III), and you must determine which of these, if any, is also a fact. 50. Fact 1: A project team consisting of males and females has four members. Fact 2: Two of the members are proficient in mathematics and the other two are proficient in computer programming. Fact 3: Half the members are female. If the first three statements are facts, which of the following statements must also be a fact? Ι . At least one female member is proficient in mathematics. Ι Ι . Two of the members are male. Ι Ι Ι . The male members are proficient in computer programming. (A) Ι Ι only (B) Ι and Ι Ι Ι only (C) Ι Ι and Ι Ι Ι only (D) No ne Solution: From facts 1 and 3, it is clear that there are two females and two males. From fact 2, we can only say that two of them are proficient in programming. But we do not know the distribution between males and females. ∴ Only Ι Ι is a fact. Choice (A) 51. Fact 1: Manoj said, "Anush and I both went to a movie last night." Fact 2:Anush said, "I was only studying last night." Fact 3: Manoj always tells the truth, but Anush sometimes lies. If the first three statements are facts, which of the following statements must also be a fact? Ι . Anush went to a movie last night. Ι Ι . Manoj went to a movie last night. Ι Ι Ι . Anush was studying last night. (A) Ι Ι only (B) Ι only (C) Ι and Ι Ι only (D) Ι , Ι Ι and Ι Ι Ι Solution: From facts 1 and 3, it is clear that Anush and Manoj went to a movie last night. From fact 2, Anush said that, he was only studying last night, which is not true. But he may or may not be studying ∴ We cannot say that Ι Ι Ι is a fact. ∴ Ι and Ι Ι are facts. Choice (C) 52. Fact 1: Chairs cost between Rs.200 to Rs. 2000. Fact 2: Some chairs are made of aluminum. Fact 3: Some chairs are made of plastic If the first three statements are facts, which of the following statements must also be a fact? Ι . Aluminum chairs cost more than plastic chairs. Ι Ι . Expensive chairs last longer than cheap chairs. Ι Ι Ι Plastic chairs costs around Rs.200 and aluminum chairs costs around Rs. 2000. (A) Ι only (B) Ι Ι only (C) Ι and Ι Ι Ι only (D) None Solution: From facts 1, 2 and 3, we do not get any relationship between the costs of aluminium and plastic chairs. Neither I nor III is a fact. Also, we cannot say the relationship between expensive and cheap chairs. ∴ Ι Ι is not a fact. Choice (D) 53. Fact 1: All metros have ring roads. Fact 2: Delhi is a metro. Fact 3: Delhi has a population of more than 5 million.

If the first three statements are facts, which of the following statements must also be a fact? Ι . Delhi has a ring road. Ι Ι . All metros have a population more than 5 million. Ι Ι Ι . All cities with a ring road are metros. (A) Ι only (B) Ι and Ι Ι only (C) Ι and Ι Ι Ι only (D) Ι , Ι Ι and Ι Ι Ι Solution: From facts 1 and 2, we can say that, Delhi, a metro has ring roads. ∴ Ι is a fact. But Ι Ι Ι may or may not be a fact. From fact 3, even though Delhi, a metro, has a population of more than 5 million, we cannot say that all metros have a population more than 5 million. ∴ Ι Ι is not a fact. ∴ Only Ι is a fact. Choice (A) Directions for Questions 54 to 56: On the basis of given two facts, determine which of the conclusions marked A, B, C or D can be most logically drawn. 54. Fact 1: Some musicians play Tabla. Fact 2: All the Tabla players need to be trained for at least 10 years. (A) Children of Tabla players may require less than 10 years of training. (B) All the musicians who have trained for at least 10 years are Tabla players. (C) Some of the musicians may have been trained for at least 10 years. (D) All the musicians are Tabla players. Solution: From fact 1 and 2, some musicians who play Tabla, must have trained for at least 10 years. ∴ Only (C) can deducted Choice (C)

be

55. Fact 1: Cloudy days tend to be windier than sunny days. Fact 2: Foggy days tend to be less windy than cloudy days. (A) Sunny days tend to be less windy than foggy days. (B) Sunny days tend to be windier than foggy days. (C) Foggy days and cloudy days tend to be windier than sunny days. (D) Foggy days and sunny days tend to be less windy than cloudy days. Solution: From fact 1, sunny days tend to be less windy than cloudy days. From fact 2, foggy days tend to be less windy than cloudy days. ∴ only follows. Choice (D)

(D)

56. Fact 1: At parking lot, a car is parked to the right of a truck and to the left of a van. Fact 2: A jeep is parked to the right of the truck. (A) The car is to the left of the jeep. (B) The jeep is to the right of the van. (C) The jeep is parked between the car and the truck. (D) The truck is to the left of the jeep. Solution: From fact 1, we have Truck Car Van From fact 2, Truck Jeep From the above two facts, we can not determine the relative position of the jeep with respect to the car and van.

∴ Only follows.

(D) Choice (D)

57. He must be an IIT student; he is wearing a shirt with the IIT logo on it. This conclusion is valid only if it is true that (A) All IIT students wear shirts with IIT logo on it. (B) IIT students never wear any shirt without IIT logo on it. (C) IIT students are required to wear shirts with IIT logo on it. (D) Only IIT students wear shirts with IIT logo on it. 58. Five people witnessed a thief leaving a house that was locked. Each gave the following description of the thief in the court. Witness 1: He was short, thin, and old. Witness 2: He was tall, thin, and young. Witness 3: He was short, stout, and young. Witness 4: He was tall, stout, and old. Witness 5: He was tall, stout, and young. Which of the following descriptions of the thief is probably correct? (A) Tall, stout and old (B) Short, thin and old (C) Short, stout and young (D) Tall, stout and young 59. In recommending a salary cut of five percent to the board of directors, the CEO of a company said: "There were no worker demonstrations over the previous salary cuts of three percent last year and two percent the year before." If the CEO's statement is accurate, which of the following can be validly deduced from the information given? Ι . Most workers in the previous years felt that the salary cuts were justified because of increased operating costs. Ι Ι . Workers apathy was responsible for the failure of the workers to protest the previous salary cuts. Ι Ι Ι . Workers are not likely to demonstrate over the new salary cuts. (A) Ι and Ι Ι Ι only (B) Ι Ι and Ι Ι Ι only (C) Ι , Ι Ι and Ι Ι Ι (D) Nei ther Ι , Ι Ι nor Ι Ι Ι 60. Ravi has scored over sixty percent marks in his High School Examination. This statement can be logically deduced from which of the following statements? (A) The average marks for the class is 60%, and his rank in the class is 46 out of 91. (B) He is admitted to a prestigious college where nobody with a second class is admitted. (C) Indira has scored less than 60% and also did not qualify for the scholarship. Ravi has been selected for the scholarship. (D) Everyone scoring less than 60% must appear for re-examination for improving their marks. Ravi is not required to appear for re-examination. Solution: From (A), we cannot get any information about the percentage of marks scored by Ravi. From (B), the “second class’ is not defined. From (C), as the cutoff for the scholarship is not given, we cannot deduce anything. From (D), as Ravi is not required to appear for re-examination, we can say that he did not get less than 60% marks in his high school examination Choice (D) 61. Four people Ahmed, Burman, Chhaya, and Deepak in that order occupy the four corners of a square of a side a in clock wise order. Ahmed and Burman simultaneously start walking at the same speed towards Burman and Chhaya respectively. Both of them stop walking when Burman reaches Chhaya. What is the distance between Ahmed and Burman? (A) a Solution:

(B) 0

(C) a

(D) > a

The directions of Ahmed is changing every moment in clockwise direction. Had Ahmed travelled along the diagonal towards Chhaya, he would have covered a distance of ‘a’ by the time Burman meets Chhaya. ∴ The distance between them would be a −a = a ( − 1). As Ahmed did not travel along the diagonal, the distance between Ahmed and Burman will be greater

than − 1).

a

(

Choice (D)

Directions for questions 62 to 67: The sentences given below when properly sequenced form a coherent paragraph. Each sentence is numbered. Select the most logical order of the sentences in each case. 62. (i) Liberalization of the aviation sector has led to the arrival of many private carriers in the domestic market. (ii) The railways have introduced some very ingenious schemes to retain their customers. (iii) The lower airfares have cannibalised the first class travellers in the railways due to the parity in fares and the reduction of travel times. (iv) This situation of having choices in the mode of transportation is of delight to the customers. (A) (i)-(iv)-(ii)-(iii) (B) (i)-(iii)-(ii)-(iv) (C) (i)-(iv)-(iii)-(ii) (D) (i)-(iii)-(iv)-(ii) Solution: Sentence (i) states the topic and begins the para. Sentence (iii) follows – the idea moves from ‘private carriers’ to ‘lower airfares’ which are the consequence sentence (ii) follows with reaction of the real ways and sentence (iv) concludes pointing to the customers and the beneficiaries. Choice (B) 63. (i) The ongoing war in Iraq is perceived by many as a serious threat to world peace. (ii) The real benefit of the war is yet to be realized, but the losers have been the people of Iraq. (iii) The supporters of the war point out that human rights violation is serious enough to have warranted the war in Iraq, in spite of the increased risk to world peace. (iv) Both who support and oppose the war have valid points to bolster their arguments. (A) (i)-(iii)-(iv)-(ii) (B) (iii)-(i)-(iv)-(ii) (C) (ii)-(i)-(iii)-(iv) (D) (ii)-(iii)-(i)-(iv) Solution: Sentence (i) states the topic – the war in Iraq. Sentence (iii) follows linked by the idea of world peace. From supporters’ in (iii) we came to those ‘who support and oppose’ in sentence (iv) and sentence (ii) concludes with the benefits and losses. Choice (A) 64. (i) The history of civilization had to be rewritten in the 19th and the 20th century after it was established that these paintings were produced by stone age dwellers. (ii) The owner of the animal rescued it, but in the process discovered those caves. (iii) Discovered in 1865, the cave paintings popularly referred to as the 'Sistine chapel' of the stone age are estimated to have been created around 12000 B.C. (iv) The discovery of the stone age paintings was made possible when a hunting dog got trapped in the cave. (A) (i)-(iii)-(iv)-(ii) (B) (iii)-(i)-(iv)-(ii) (C) (iv)-(ii)-(i)-(iii) (D) (iii)-(iv)-(ii)-(i) Solution: The words ‘trapped on the cave on (iv) leads to ‘rescued’ in sentence (ii) (iv) and (ii) go together. We find this in choices C and D. Clearly (i) follows (iv)-(ii). Sentence (iii) is better at the beginning since it introduces the ‘cave painting’ – the subject of the para. Choice (D) 65. (i) Aggressive play may not be instigated in capacity because the development of hunting skills is irrelevant in captivity. (ii) However, tiger cubs born in captivity never engage in aggressive play. (iii) The cubs' parents generally instigate the aggressive play between the cubs. (iv) Young tiger cubs in the wild are often found to engage in aggressive play with their siblings. (A) (i)-(ii)-(iv)-(iii) (B) (i)-(ii)-(iii)-(iv) (C) (iv)-(iii)-(i)-(ii) (D) (iv)-(iii)-(ii)-(i) Solution:

Sentences (ii) and (i) go together ‘never engage in aggressive play’ in (ii) heads to ‘is irrelevant in captivity in (i) sentence (iv) begins by stating the subject – ‘aggressive play’ in tiger cubs. This is followed by the role of the cubes parents. Choice (D) 66. (i) If a new list of the world's wonders was necessary, it should have been compiled by UNESCO, and not by any private organization. (ii) The electronic media had relentlessly campaigned for the cause of Taj Mahal, motivating all the Indians to vote for the monument so that it would be included in the new list of the seven wonders of the world. (iii) The Taj does not require any campaign to prove its timeless beauty. (iv) It was a totally unnecessary campaign. (A) (ii)-(i)-(iv)-(iii) (B) (ii)-(i)-(iii)-(iv) (C) (ii)-(iv)-(iii)-(i) (D) (i)-(ii)-(iii)-(iv) Solution: Sentence (ii) clearly beings the para. The words ‘relent-lessly campaigned’ in (ii) leads to ‘totally unnecessary campaign in (iv). This is followed by (iii) – the Taj does not require any campaign sentence (i) winds up with who if any one should have organized the campaign. Choice (C) 67. (i) It is more than a budgetary move for the Nepal's interim government to strip the royal family of its annual allowance. (ii) To make the monarchy irrelevant is the next logical step forward for the government. (iii) Monarchs' do not draw all their powers from their purses; though they do not like being deprived of the funds. (iv) To carry the palace's expenses is a big burden for a small country. (A) (i)-(iii)-(iv)-(ii) (B) (iii)-(iv)-(i)-(ii) (C) (iii)-(i)-(iv)-(ii) (D) (i)-(iii)-(ii)-(iv) Solution: Sentence (i) states the topic – the intention of the Nepoli government. The royal family stopped of its annual allowance leads to (iii) – monarch’s do not draw all their power from money but they do not like to be deprived of it. Sentence (ii) is clearly the concluding line the real intention of the government. Choice (A) 68. If you get a JMET rank of better than 500, then you will get an admission in your preferred institute. If the statement above is true, which of the following must also be true? Ι . If you do not get a JMET rank of better than 500, then you will not get an admission in your preferred institute. Ι Ι . If you get an admission in your preferred institute, then you must have got a JMET rank of better than 500. Ι Ι Ι . If you did not get an admission in your preferred institute, then you did not get a JMET rank of better than 500. (A) Ι Ι only (B) Ι Ι Ι only (C) Ι and Ι Ι Ι only (D) Ι , Ι Ι and Ι Ι Ι Solution:

P

If yo u get then a Q Yo JM u ET will ran get kan of ad bet mis ter sio The implications are of the form tha n (1) P Q n in 50 yo (2) ~ Q ~P 0 ur pre ferr ed inst itut e

∴ Only follows

III Choice (B)

Directions: Questions 69 to 72 are based on the following passage: Amit, Balvinder, Chetan, and Deepak are employed in a company, where they have to share among themselves the work load that consists of six tasks A, B, C, D, E and F. The following statements identify their preferences for the different tasks. All those who like task B also like task E. All those who like task C also like task D. All those who like task E do not like task C, and vice-versa. Some of those who like task E also like task A. Some of those who like task D also like task E. All those who like task D also like task F. 69. Amit enjoys the task D. Which of the following must be true? (A) He may or may not like the task C. (B) He does not like the task B. (C) He likes the task A. (D) He likes the task C. 70. Balvinder likes the task B. He may also like any of the following tasks, except (A) Task A (B) Task C (C) Task D (D) Task E 71. Chetan likes the task C. Which of the following must be false? (A) He does not like the task A. (B) He likes the task F. (C) He does not like the task B. (D) He may like the task E. 72. Based on the information provided, which of the following statements must be true? (A) All of those who like the task E also like the task B. (B) None of those who do not like the task F like the task A. (C) Those who like the task A may or may not like the task C. (D) None of those who like the task B do not like the task D. Directions: Questions 73 to 78 are based on the following passage: There are six blocks of rooms along a straight corridor in a hotel with each block containing two rooms facing each other.

LEFT Corridor

RIGHT

The following gorup of twelve people, Jitender, Lakshman, Mary, Narayan, Pankaj, William, Chandra, Ahmed, Balu, Ferosh, Esha and Rajender has occupied some of these rooms. There are a maximum of two people in a room, and some rooms may be empty. Lakshman and his roommate stay two blocks to the right of Ahmed and his roommate Chandra. Jitender stays alone, three blocks to the left of William and two blocks to the left of Esha. Mary stays one block to the left of Ahmed and Chandra. Narayan stays three blocks to the right of the block on which Balu and Ferosh have single rooms. Rajender and Pankaj stay in single rooms two blocks to the left of Mary. 73. Which of the following lists the persons in the correct order, going from the left most block to the right? (A) Rajender, Balu, Mary, Ahmed, Lakshman, Narayan (B) Rajender, Ferosh, Narayan, Esha, Lakshman, Chandra

(C) Pankaj, Balu, Jitender, Chandra, Narayan, Lakshman (D) Lakshman, Esha, Ahmed, Mary, Ferosh, Rajender Solution: Let each person be denoted by the first letter of his/her name. From the first, third and the fifth statements, we have 1 2 R P From the second statement we have

3

4

M

AC

J

E

5

6 L

W

From the fourth statement we have, B F

N

As in the first block, R and P are in single rooms and also B and F should be in the same block with single rooms, B and F cannot be in the first block and in the third block [∵ M stays in the 3rd block]. ∴ B and F should be in the 2nd block. ∴ N is in the 5th block. As already, R, P, M, A, C, B, F and N are staying in the first five blocks, one among J, E and W must be L’ s room mate. ∴ He must be W. ∴ We have the following final distribution. 1 R P

2 B F

3 M J

4 AC

5 NE

6 LW

M and J stays. As J stay In the 3rd block alone, they stay in single rooms. In the 5th block, N and E stay in either single rooms or in one room. Only order.

(C)

is

a

correct

Choice (C)

74. Which of the following pairs must stay on the same block? Ι . Narayan and Esha Ι Ι . Jitender and Mary Ι Ι Ι . Ahmed and Lakshman (A) Ι only (B) Ι Ι Ι only (C) Ι and Ι Ι only Ι Ι Ι only Solution: Let each person be denoted by the first letter of his/her name. From the first, third and the fifth statements, we have 1 2 R P From the second statement we have J

3

4

M

AC

E

5

6 L

W

(D) Ι Ι and

From the fourth statement we have, B F

N

As in the first block, R and P are in single rooms and also B and F should be in the same block with single rooms, B and F cannot be in the first block and in the third block [∵ M stays in the 3rd block]. ∴ B and F should be in the 2nd block. ∴ N is in the 5th block. As already, R, P, M, A, C, B, F and N are staying in the first five blocks, one among J, E and W must be L’ s room mate. ∴ He must be W. ∴ We have the following final distribution. 1 R P

2 B F

3 M J

4 AC

5 NE

6 LW

M and J stays. As J stay In the 3rd block alone, they stay in single rooms. In the 5th block, N and E stay in either single rooms or in one room. The pairs N and E, J and M, R and P, B and F, A and C, L and W stay in the same block. Choice (C) 75. Lakshman's roommate, assuming that he or she is one of the persons mentioned, is (A) Esha (B) William (C) Mary (D) Narayan Solution: Let each person be denoted by the first letter of his/her name. From the first, third and the fifth statements, we have 1 2 R P From the second statement we have J

3

4

M

AC

E

5

6 L

W

From the fourth statement we have, B F

N

As in the first block, R and P are in single rooms and also B and F should be in the same block with single rooms, B and F cannot be in the first block and in the third block [∵ M stays in the 3rd block]. ∴ B and F should be in the 2nd block. ∴ N is in the 5th block. As already, R, P, M, A, C, B, F and N are staying in the first five blocks, one among J, E and W must be L’ s room mate. ∴ He must be W. ∴ We have the following final distribution. 1 R P

2 B F

3 M J

4 AC

5 NE

6 LW

M and J stays. As J stay In the 3rd block alone, they stay in single rooms. In the 5th block, N and E stay in either single rooms or in one room. W mate.

is

L’s

room

Choice (B)

76. Rajender stays on the (A) first block, and to the left of Balu or Ferosh. (B) second block, and to the left of Jitender or Ahmed and Chandra. (C) third block, and to the right of Mary or Esha. (D) fourth block as Ahmed and Chandra. Solution: Let each person be denoted by the first letter of his/her name. From the first, third and the fifth statements, we have 1 2 R P From the second statement we have

3

4

M

AC

J

E

5

6 L

W

From the fourth statement we have, B F

N

As in the first block, R and P are in single rooms and also B and F should be in the same block with single rooms, B and F cannot be in the first block and in the third block [∵ M stays in the 3rd block]. ∴ B and F should be in the 2nd block. ∴ N is in the 5th block. As already, R, P, M, A, C, B, F and N are staying in the first five blocks, one among J, E and W must be L’ s room mate. ∴ He must be W. ∴ We have the following final distribution. 1 R P

2 B F

3 M J

4 AC

5 NE

6 LW

M and J stays. As J stay In the 3rd block alone, they stay in single rooms. In the 5th block, N and E stay in either single rooms or in one room. R stays F. Choice (A)

on

the

first

block

and

to

77. An empty room or empty rooms may be found in the (A) second block only. (B) fourth block only. (C) third or sixth block, but not both. (D) fourth or sixth block or both. Solution: Let each person be denoted by the first letter of his/her name. From the first, third and the fifth statements, we have

the

left

of

B

and

1 2 R P From the second statement we have

3

4

M

AC

J

E

5

6 L

W

From the fourth statement we have, B F

N

As in the first block, R and P are in single rooms and also B and F should be in the same block with single rooms, B and F cannot be in the first block and in the third block [∵ M stays in the 3rd block]. ∴ B and F should be in the 2nd block. ∴ N is in the 5th block. As already, R, P, M, A, C, B, F and N are staying in the first five blocks, one among J, E and W must be L’ s room mate. ∴ He must be W. ∴ We have the following final distribution. 1 R P

2 B F

3 M J

4 AC

5 NE

6 LW

M and J stays. As J stay In the 3rd block alone, they stay in single rooms. In the 5th block, N and E stay in either single rooms or in one room. Empty rooms are in 4th and 6th blocks. In the 5th block these may or may not be empty rooms. Choice (D) 78. Jitender arranges to move into a room two blocks to the left, whose occupant moves into a room one block to the right. In turn, the occupant of this room moves into a room three blocks to the right, whose occupant takes Jitender's old room. The new occupant of Jitender's old room is (A) Balu or Ferosh (B) Narayan or Esha (C) Mary (D) Rajender Solution: Let each person be denoted by the first letter of his/her name. From the first, third and the fifth statements, we have 1 2 R P From the second statement we have J

3

4

M

AC

E

5

L

W

From the fourth statement we have, B F

6

N

As in the first block, R and P are in single rooms and also B and F should be in the same block with single rooms, B and F cannot be in the first block and in the third block [∵ M stays in the 3rd block]. ∴ B and F should be in the 2nd block. ∴ N is in the 5th block. As already, R, P, M, A, C, B, F and N are staying in the first five blocks, one among J, E and W must be L’ s room mate. ∴ He must be W. ∴ We have the following final distribution. 1 R P

2 B F

3 M J

4 AC

5 NE

6 LW

M and J stays. As J stay In the 3rd block alone, they stay in single rooms. In the 5th block, N and E stay in either single rooms or in one room. Given J moves to block 1. A person from block 1 moves to block 2. A person from block 2 moves to block 5. A person from block 5 moves to J’s old room. ∴ A person from block 5 i.e., either N or E, moves to J’s old room. Choice (B) 79. The following information is given about a four sided polygon. Ι . The polygon is a rectangle.' Ι Ι . The area of the polygon is given to be 100 m2. Ι Ι Ι . One side of the polygon is 8 m. Ι V. All the adjacent sides are at right angle to each other. Which of the above facts are sufficient to determine the dimensions of the polygon? (A) II and III (B) II, III and IV (C) I, III and IV (D) I and II Solution: (A) From II and III we have, the area of the polygon and one of the sides of the polygon. But we don’t know the number of sides or the shape of the polygon. ∴ We cannot find the dimensions. (B) From II, III and IV, we have the area, one of the sides and angle between all the adjacent sides as 90o which means the polygon is a rectangle. ∴ We can find the dimensions of the polygon as 8 and 12.5. ∴ From (B) we can answer the questions. (C) From I, III and IV, we know only one side of the polygon and also the polygon is a rectangle. ∴ We cannot find the dimensions of the polygon. (D) From I and II, we can only say the area of the polygon but not length and breadth. Choice (B) 80. Students rank the business schools based on the following factors: Average salary of a fresh graduate, student to faculty ratio, terminal degree of faculty members, and institutional facilities. In their final ranking school A is ranked higher than school B? (A) The average salary of fresh graduates from school A is 60% less than the average salary of fresh graduates from school B. (B) The average salary of fresh graduates from school A is 80% more than the average salary of fresh graduates from school B. (C) All the faculty members in school A have a doctoral degree while in school B only 50% of the faculties have a doctoral degree. (D) In all the factors school A is marginally better than school B.

Section 3: QUANTITATIVE ABILITY (81 – 120)

Directions: Questions 81 to 84 relate to the AIRCRAFT PURCHASE problem given below: India's national airline 'INDIAN' is considering purchase of new aircraft to meet an estimated demand of 4600 seats. The table below presents the relevant data.

Aircraft

Seats

Price ($ millions)

B747

400

250

B777

300

150

A321

200

100

It has planned to buy three B777's and five A321's for every B747. 81. How many of each aircraft should the company purchase? (A) B747 = 2; B777 = 6, A321 = 10 (B) B747 = 4; B777 = 6; A321 = 6 (C) B747 = 3; B777 = 8; A321 = 5 (D) B747 = 1; B777 = 6; A321 = 12 Solution: From choice (A), the total number of seats that will be available after the purchase is 2(400) + 6(300) + 10(200) = 4600, which is same as the estimated demand. Choice (A) 82. However, INDIAN's budget is limited to $2000 million for this purchase. Given this constraint, it is willing to be flexible on the proportion of aircraft types to be purchased. How many aircraft should it purchase such that both budget utilization and meeting the estimated seat demand are simultaneously maximized? (A) B747 = 2; B777 = 4; A321 = 9 (B) B747 = 2; B777 = 6; A321 = 6 (C) B747 = 2; B777 = 5; A321 = 7 (D) B747 = 1; B777 = 5; A321 = 10 Solution: From Choice (A) the total, expenditure incurred will be 2(250) + 4(150) + 9(100) = 2000 The total number of seats available in million $ is 2(400) + 5(300) + 7(200) = 3700 Similarly the expenditure incurred and seats available for all the four options are tabulated below Expenditure Seats Choice (A) 2000 3700 Choice (B) 2000 3800 Choice (C) 1950 3700 Choice (D) 2000 3900 Choice (D) 83. How many different alternatives for aircraft purchase are possible for full use of the budget? (A) 5 (B) 6 (C) 3 (D) 4 Solution: Let the number of the B747, B777 and A321 be x,y,z, respectively ∴ 250x + 150y + 100z = 2000 ⇒ 5x + 3y + 2z = 40 We see that 0 ≤ x ≤ 8 0 ≤ y ≤ 13 0 ≤ z ≤ 20 If x = 0 3y + 2Z = 40 and (y, z) = (0.20), (2,17)--- (12,2) i.e., there are 7 values for (x, y, z) If x = 1, 3y + 2z = 35 and (y, z) = (1,16), (3,14) ------ (11,1) If there are 6 values for (x, y, z)

If x = 2, there are more values We see that there are many more than 6 values for (x, y, z) The correct answer is not there among the options. 84. If a deviation of at most $50 million below the budget is permitted, how many additional alternatives for aircraft purchase are possible? (A) 5 (B) 3 (C) 4 (D) 6 Solution: Without any deviation, there are many more then 6 alternatives for the number of aircrafts. With the permissible deviation of $50 million there will be more. The correct answer is not there Directions: Questions 85 and 86 relate to the DISTRIBUTION CHANNEL problem given below: There are two markets for selling a product, and two channels for distribution of the product to these markets. The cost of distribution per unit of the product through Channel 1 and Channel 2 are f1 and f2 respectively. The total cost incurred by the seller for Market 1 and Market 2 are F 1 and F2 respectively. Let q1and q2 be the quantities of product distributed to Market 1 and Market 2 through Channel 1. Let q 3 and q4 be the quantities of product distributed to Market 1 and Market 2 through Channel 2. 85. Which of the following linear equations in matrix form correctly represents this problem?

(A)

(B)

(C)

(D) Solution: All the 4 questions show the F1, the cost for market1 at the top and F2 the cost for market 2 at the bottom. The symbols q1 and q2 are associated with channel 1 and q3 and q4 with channel 2. The symbol f1 is associated with channel1 and f2 with channel 2. We represent this in the diagram below.

From this, we can write down the two equations. f1q1 + f2q3 = F1 ……(1) f1q2 + f2q4 = F2 ……(2)

(1) & (2) can be expressed in matrix form as Choice (D) 86. Given q1 = 2, q2 = 4, q3 = 4, q4 = 8, F1 = 10, and F2 = 20, which of the following is true? (A) f1 = 1 and f2 = 2 is one of the possible solutions (B) f1 = 1 and f2 = 2 and it is unique (C) f1 = 2 and f2 = 1 and it is unique (D) f1 = 2 and f2 = 1 is one of the possible solutions Solution: We have, f1q1 + f2q3 = F1 → (1) f1q2 + f2q4 = F2 → (2) From the given values we get 2f1 + 4f2 = 10 [from (1)] 4f1 + 8f2 = 20 [from (2)] the two equations are equivalent and they have more than one solution. f 1 = 1 and f2 = 2 is one of the possible solutions. Choice (A) Directions: Questions 87 to 89 relate to the locations of WAREHOUSE AND SUPERMARKETS given below: A retail major has a warehouse (W) located at (16, 10) in a town having roads laid on a square grid parallel to the x and y axes. There are five retail supermarkets (M, N, O, P and Q) located respectively at (4, 4), (6, 16), (16, 24), (20, 16) and (26, 4) 87. What is the ordering of the supermarkets from the nearest to the farthest from the warehouse? (A) P, M, N and Q, O (B) P, N and Q, O, M (C) P, O, N and Q, M (D) P, O, M, N and Q Solution: The location of the warehouse (W) and the 5 super markets are as follows. W (16, 10) M (4, 4) O (16, 24) P (20, 16) Q (26, 4) We need to order the markets from the nearest to the farthest. We should consider not the straight line distance, but the distance along the roads. WM = (16 – 4) + (10 – 4) = 18 Similarly, WN, WO, WP, WQ are 16, 14, 10,16. ∴ The required order in P, O, N/Q, M, Choice (C) 88. Suppose each square block in the grid has sides of length 2 km. The minimum length of a round trip starting from M and moving through N, O, P, Q and returning to M will be (A) 84 km (B) 42 km (C) 80 km (D) 40 km Solution: The (minimum) length of the roundtrip MNOPQM is MN + NO + OP + PQ + QM = 14 + 18 + 12 + 18 + 22 = 84 As the side of each square block in the grid is 2 km, we should expect 168 km among the options. This is not there Note: The candidate may want to select choice A, under the given circumstances. Choice (A)

89. Five trucks are used, one each to travel from the warehouse to the supermarkets M, N, O, P and Q. Suppose their average speeds are respectively 54, 50, 42, 25 and 40 kmph. Assume that the trucks are identical and their drivers have identical driving skills and styles. If five trucks start simultaneously from the warehouse, which truck will reach its destination the earliest? (A) W to M (B) W to O (C) W to P (D) W to N Solution: The distances that the 5 trucks cover and their speeds are shown below

Dist Speed

WM

WN

NO

WP

WQ

18

16

14

10

16

54

50

42

25

40

Time

first.

In the least time, i.e., the truck going from W to N reaches its destination Choice (D)

Directions: Questions 90 to 92 relate to the EXCHANGE RATES problem given below: The official 'buy' and 'sell' exchange rates for the US $, UK £, and EU €, with reference to the Indian INR are presented in the table below: Bank BUYS

Bank SELLS

You sell

Bank Pays You

You Pay bank

You buy

$1

INR 40

INR 42

$1

£1

INR 77

INR79

£1

€1

INR60

INR 62

€1

Your local bank agrees to sell $0.023 or £0.012 or €0.015 for INR 1. 90. You wish to buy foreign currency with INR 1lakh. Based on the values of each foreign currency you will receive from the bank, arrange them in the descending order? (Please note: 1/42 = 0.024; 1/79 = 0.013; 1/62 = 0.016) (A) $ > € > £ (B) € > £ > $ (C) £ > € > $ (D) all are equal Solution: From the given information if we buy $1 and then convert it into INR we loose INR 2. Similarly if we buy ₤ 1 and convert into INR we loose INR 2 and if we buy € 1 and then convert it into INR we loose INR 2. Hence for all the three currencies the loss is the same. But in order to buy a ₤ we have to spend more INR than for a €, which, in twin is more than that for a $ Hence the loss will be minimum (i.e., value will be maximum) for ₤ followed by € and $. Choice (C) 91. When you buy foreign currency from your local bank, it will levy a transaction fee equivalent of INR 500 and an additional INR 500 to deliver the exchanged money to the branch of your choice. This total

amount of INR 1000 will be deducted from the foreign currency payable to you. At the airport, the money changer is willing to offer $0.022 for INR 1. What is the range of values of INR that can be exchanged for buying the $ which will get you a better deal at the airport than the bank? (A) INR 0 to 1000 (B) INR 1000 to 23000 (C) INR 0 to 23000 (D) INR > 23000 Solution: Let us suppose that INR × can be exchanged for buying the $ at the airport. The number of $ that we get at the airport is $ × (0.022) The number of $ that we get from the local bank for INR x is $ × (0.023). But from this a total INR 1000 i.e., $23 is deducted by the bank. Hence we get $× (0.023) – $23. The deal at the airport will be better if × (0.022) ≥ × (0.023) – 23 ⇒ x(0.001) ≤ 23 i.e., x ≤ 23000 Choice (C) 92. The £ to € 'buy' rate is £ 1 = € 1.222. Using the INR as the reference currency, determine by what percentage this 'buy' rate should change such that there is no arbitrage (or, differences among the three pairwise exchange rates) across the three currencies? (A) approx.5% (B) approx. 36% (C) approx. 4.25% (D) approx. 35.75% Solution: The question refers to the ‘you buy’ rates ⇒ £ 1 = Rs.79 and € 1 = Rs.62

∴ £1=

= € 1.274

While the given rate in £ 1 = € 1.222

∴ This rate should increase by

= %

(100%) ≈ 4.25 Choice (C)

Directions: Questions 93 and 94 relate to the BPO HR FLOWS problem described below: Three BPOs, X, Y and Z have 500, 650 and 800 permanent employees respectively on 1st January 2006. The table below provides data on the average number of employees who quit one BPO and join another in a month. TO → FROM ↓

X

Y

Z

X

0

5

3

Y

8

0

1

Z

10

12

0

The second table below provides data on the retirements and retrenchments from X, Y and Z (these people are not re-employed in any of these three companies), and the additional fresh recruitments made by the three BPOs per month. X

Y

Z

Retirements and Retrenchments

3

6

10

Fresh Recruits

10

12

20

All the joining or leaving events happen at the end of each month. 93. What will be employee strengths of the three companies on 31st December 2006? (A) X = 704; Y = 818; Z = 704 (B) X = 687; Y = 804; Z = 712 (C) X = 610; Y = 738; Z = 602 (D) X = 620; Y = 746; Z = 584 Solution: Given the number of employees in X, Y and Z is 500, 650 and 800 respectively as on 1st Jan 2006. All joining or leaving events happen at the end of each month. In order to calculate the strengths of the three companies on 31st Dec, 2006, we have to consider 11 months and not 12 months. Hence the strengths of X, Y and Z will be 500 + 11(17), 650 + 11(14) and 800 – 11(8) i.e., 687, 804 and 712 Choice (B) 94. In which month will be sum of the absolute value of differences in employee strengths between X and Y and Z be the least? (A) June (B) July (C) August (D) September Solution: Let the sum of the absolute value of the differences in employee strengths between X, Y and Y, Z be the least after ‘n’ months. (n ≤ 12). Clearly Y > X for any value of ‘n’ Now, Z > Y i.e., 800 – 8n > 650 + 14n if n ≤ 6 (i.e., until June) Similarly Z < Y, if n ≥ 7 Suppose Z > Y (i.e. n ≤ 6) Now, S = [650 + 14n – (500 + 17n) + [800 – 8n – (650 + 14n)] i.e., S = (150 – 3n) + (150 − 22n) i.e., S = 300 – 25n Now ‘S’ will be minimum if n is maximum i.e. n = 6 ∴ Min S = 300 – 25(6) = 150 → (1) Suppose Z < Y (i.e n ≥ 7) (After June) Now S = [650 + 14n – (500 + 17n)] + [650 + 14n – (800 – 8n)] i.e., S= 150 – 3n – 150 + 22n = S – 19n. Now ‘S’ will be minimum if n is minimum i.e., n = 7 ∴ Min S = 19(7) = 133 → (2) From (1) and (2) minimum value of S is 133 for n = 7 i.e., by the end of July the value of ‘S’ will be minimum. ∴ ‘S’ will be minimum in the month of August. Choice (C) Directions: Questions 95 and 96 relate to the STATIONERY MANUFACTURING problem given below:

A stationery manufacturer produces two pen varieties, standard (S) and premium (P). These are packed and sold respectively at Rs.10 and 25. Two machines, M1 and M2, are available to produce both varieties. M1 can produce 30 and 50 pens (including refills) per minute of S and P respectively. M2 can produce 40 an 60 per minute of S and P respectively. The manufacturer runs two shifts of 8 hours each. 95. The above partial information can then be formulated mathematically as (A) Max Z = 10S + 25P Subject to 71S ≤ 960 110P ≤ 960 S, P ≥ 0, and integers (B) Max Z = 10S + 25P Subject to: S/70 ≤ 960 P/110 ≤ 960 S, P ≥ 0, and integers (C) Max Z = 10S + 25P Subject to: S/30 + P/50 ≤ 960 S/40 + P/60 ≤ 960 S, P ≥ 0, and integers (D) Max Z = 10S + 25P Subject to: 30S +50P ≤ 960 40S + 60 P ≤ 960 S, P ≥ 0, and integers Solution: The manufacturer produces two varieties of pens – standard and premium. Let S be the number of standard pens and P be the number of premium pens manufactured. Given that the selling price of each standard pen is Rs.10 and that of premium pens is Rs.25. Let Z be the total revenue from their sales. ∴ Z = 10S + 25P. It is given that two machines M1 and M2 are used to manufacture these pens and each machines works for a maximum of 8 hrs. per shift Also M1 produces 30 standard and 50 premium varieties of pens in one minute.

∴ Also M2 produces 40 standard and 60 premium varieties of pens in one minute.

∴ Since the revenue is to be maximum, we can represent this entire data as Max Z = 10S + 25P Subject to:

Also integers.

S,

P

are

non-negative

Choice (C) 96. The manufacturer also ensures that at any time at least 80 more and 50 more of refills (R) are produced for S and P respectively. The same refill is used for S and P. This constraint can be specified as (A) R = 130

(B) R – (S + P) ≥ 130 (C) R = 80 + S, R = 50 + P (D) R ≥ 130 Solution: Let R,, and R2 be the number of extra refills produced for standard and premium variety of pens respectively Given R1 ≥ 80 R2 ≥ 50 Also given that the total number of refills R = S + P + R1 + R2 i.e., R ≥ S + P + 130 ⇒R – (S + P) ≥ 130 Choice (B) 97. The change in sales volume 'S' of a new mobile phone following an advertisement campaign is given by (dS/dt) = 0.04 (700 – S). At time t = 0, S = 0. then S expressed as a function of t would be (A) S = 700 – e–0.04t (B) S = (1 – e–0.04t) / 700 (C) S = e–0.04t / 700 (D) S = 700 (1 – e0.04t) Solution:

= 0.04dt

∴ ℓn = 0.04t + k Where k is a constant When t = 0, S = 0 ∴ k = ℓn 700 i.e., ℓn i.e., e 0.04t)

= 0.04t + ℓn 700 or 700 – S = e0.04t + ℓn 700 = 700 e0.04t S

=

700

(1



Choice (D)

Directions: Questions 98 to 100 relate to the CAMPUS PLACEMENT description given below: Three companies, Muck-In-Sea (MIS), Cold Man's Axe (CMA), and Bark Laze Bank (BLB) are scheduled, in that order, to interview 8 young MBA wizards at Hogwarts for offering career placements on Day minus 1 5/8. Each company can select at most 3 students. Once a students receives an offer from a company, that students is not allowed to appeared in any more interviews. 98. How many possible combinations of student selections are there for MIS? (A) 4 (B) 93 (C) 92 (D) 3 Solution: Since the company MIS is the first company to recruit, it can select either 0 (or) 1 (or) 2 (or) 3 students out of the 8 available. Hence the possible combinations of student selections are 8C0 + 8C1 + 8C2 + 8C3 = 93 Choice (B) 99. What is the probability that CMA also does not select any student if MIS does not select any student? (A) 1/4 (B) 1/3 (C) 1/92 (D) 1/93 Solution:

Given that MIS did not select any student. This can be done in one way. Now the total number of possible combinations of student selections for CMA is8C0 + 8C1 + 8C2 + 8C3 i.e. 93. Out of which there is one way of not selecting any student.

Hence

the

required

probability

is

Choice (D) 100. Considering all the options (0, 1, 2 or 3 students) exercised by MIS and CMA, how many options does BLB have to make its selection? (A) 3 (B) 4 (C) 63 (D) 64 Solution: MIS can select 0,1,2,3 or 3students for each such option, CMA can select 0,1,2,3 or 3 students. If MIS and CMA, each select 3 students, BLB has only 3 options. For other combinations of options exercised by MIS and CMA, BLB would have all the 4 options, i.e., 0,1,2 or 3 students BLB is sure of having at least 3 options Choice (A) 101. A worker working under a bonus scheme saves 10 hours in a job for which the standard time is 60 hours. A bonus of 10% of the hourly rate is payable when he reaches 100% efficiency. An additional bonus of 2% of the hourly rate for each 1% in excess of 100% efficiency is given. If the normal wage rate is Rs.2 per hour, find the wages payable to the worker. (A) Rs.120 (B) Rs.130 (C) Rs.150 (D) None of the above Solution: The standard time for the work is 60 hrs. If a person completes it in 60 hours, we would say his efficiency is 100%. If he takes 30 hours his efficiency would be 200%. As he takes only 50 hours, i.e., 5/6 the standard time, his efficiency is 6/5 or 120% (The additional bonus is 40% of basic wage) The 3 parts of his wage are as follows: Normal wages : (50) 2 = 100 Bonus for reading 100% efficiency) : 10% (100) = 10 Additional Bonus : 40% (100) =40 Total : 150 Choice (C) Directions: Questions 102 to 105 relate to the HOSPITAL SERVICES SURVEY given below: You have conducted a survey among patients in a large hospital and developed a 'dissatisfaction index' (D)) for: (a) patients who are waiting for consultation with doctors, DIwc, and (b) patients who are in consultation with the doctors DIic. Consider that there are only two patients in the system. At time t = 0, one goes in for consultation, and the other begins waiting. Using the survey data, you have developed the following two mathematical models for representing the dissatisfaction among the two patients as a function of time 't': DIwc = (t2/75) + 0.1t DIic = 10 – 0.5t 102. What is the optimal time 't' (in minutes) that a doctor should spend offering consultation to the patient such that the total dissatisfaction of the two patients is minimized? The 'two indices are additive. (A) 12.75 mins (B) 0 mins (C) 15 mins (D) 10 mins

Solution: Given that

DIwc = DIic = 10 – 0.5t We have to find the value of ‘t’ at which DIwc + DIic is minimum

must be minimum



must be minimum

For the above quadratic expression the minimum value occurs at

t

= Choice (C)

103. What is the value of the total dissatisfaction index at this point? (A) 7 (B) 7.07 (C) 10 (D) 7.33 Solution: Given t = 15

DIwc = DIic = 10 – 0.5(15) = 2.5 ∴ Total dissatisfaction index is 4.5 + 7 Choice (A)

2.5

=

104. At what point of time will a patient waiting for consultation have the same DI as a patient in consultation? (Use 0.891/2 = 0.94) (A) approx. 15 mins (B) approx 12.75 mins (C) approx 10 mins (D) approx. 7.5 mins Solution: Given that DIwc = DIic

i.e.

⇒ ⇒ t2– + 45t – 750 = 0

∴ t=

t= i.e. t ≈ 12.75

(Since t > 0)

Choice (B)

105. Given the following functions of time t, for any value of the parameters a and b, rank them in the ascending order of function values as t → ∞? 1. f1 (t) = a2bt 2. f2(t) = a + bt 3. f3(t) = aebt 4. f4(t) = at2 + bt (A) f3(t), f1(t), f4(t), f2(t) (B) f4(t), f1(t), f3(t), f2(t) (C) f1(t), f3(t), f4(t), f2(t) (D) f2(t), f4(t), f1(t), f3(t) Solution: Among the given functions f2(t) is a linear function of t, f4(t) is a quadratic function of t, whereas f1(t) and f3(t) are exponential functions of t. As t → ∞, the value of exponential functions of t will be more than that of quadratic functions of t which will in turn be greater than that of linear function of t. Also in functions f1(t) and f3(t), the base of f3(t), is greater than that of f1(t) Hence f3(t) > f1(t) > f4 (t) > f2(t) i.e. f2(t) < f4(t) < f1 (t) < f3 (f) (as t → ∞) Choice (D) 106. The buying cost of a machine is Rs.16000. It has an estimated life of 5 years. Using the double-declining balance method of the second year? (A) Rs.9600 (B) Rs.12600 (C) Rs.3456 (D) Rs.5760 Solution: Buying cost = Rs.16,000 Lifespan = 5 years i.e., It depreciates at 20% p.a. Using the double depreciation method, it depreciates at 40% p.a. i.e., i.e., Book value at the end of two years

= 16,000

= Rs.5760 Choice (D) Directions: Question 107 and 108 relate to the data pertaining to TOURIST ARRIVALS IN KERALA given below: The following table shows the tourist arrivals in Kerala, in three different years. Assume that fluctuations in the data are random. Year 't'

1998

2002

2006

Tourists N(t) (Millions)

4.9

7.0

6.5

107. Which of the following functions of time 't' will best fit the given data? p. N(t) = a1 + b1t q. N(t) = a2 t2 + b2t + c r. N(t) = dmt s. N(t) = kent (A) r (B) s (C) q (D) p Solution: We can plot the 3 points, i.e. the number of tourists and the year as shown below.

We can see that the best fit would be a quadratic expression, i.e., the function q Choice (C) 108. Assuming that no unusual events (bird flu, chickungunya, terrorist attacks, etc) will occur over the next eight years, and that normal conditions for tourism exist, which one of the above four models would you choose for forecasting tourist arrivals two periods ahead? (A) q (B) p (C) s (D) none Solution: The quadratic expression is the best fit for the given points. But it need not be the best model to forecast future trends. We have no reason to believe that any given model can be used in this way Choice (D)

109. The return levels and associated probabilities of two securities are given below:

Security X

Security Y

Return (%)

9

12

15

Prob

0.3

0.4

0.3

Return (%)

12

15

16

Prob

0.7

0.2

0.1

What are the respective expected values and variances of returns? (A) X: E(X) = 12 and V(X) = 0 Y: E(Y) = 13 and V(Y) = 2.4 (B) X: E(X) = 12 and V(X) = 5.4 Y: E(Y) = 13 and V(Y) = 2.4 (C) X: E(X) = 12 and V(X) = 3 Y: E(y) = 14.3 and V(Y) = 4.13 (D) X: E(X) = 12 and V(X) = 4.5 Y: E(Y) = 14.3 and V(Y) = 6.19 Solution: Expected return for security x (in percentage points) is

E(x) = = 2.7 + 4.8 + 4.5 = 12 Similarly, E(y)

= = 8.4 + 3.0 + 1.6 = 13 The variance of X V(X) = (9 – 12)2(0.3) + (12 – 12)2(0.4) + (15 – 12)2 (0.3) = 9 (0.3) + 9 (0.3) = 5.4 Similarly, V(Y) = (12 –13)2(0.7) + (15 – 13)2 (0.2) + (16 – 13)2 (0.1) = 0.7 + 4(0.2) + 9(0.1) 2.4 Choice (B)

=

Directions: Read the BISCUIT PACKING problem below and answer Questions 110 to 116: A biscuit company makes cream biscuits that have 5 diameter and 0.5 cm thickness. It markets these biscuits in cylindrical packets of 5, 10, and 20 pieces each. The gaps between biscuits as well as between the packing and the biscuits are negligible. The ends of each packet are sealed with cardboard caps that have the company's logo printed on them. In your calculations, use π = 22/7. 110. The total surface areas of the above three types of packets will have the following proportion: (A) 1 : 2 : 2.5 (B) 1 : 2 : 3 (C) 1 : 1.5 : 2 (D) 1 : 1.5 : 2.5 Solution: The diameter of each biscuit is 5cm and its thickness 0.5 cm. These biscuits are packed in cylindrical packs of 5, 10 and 20 biscuits each. The radius of each of the pack will be same i.e., 25 cm.

The heights of the cylindrical packs containing 5 biscuits, 10 biscuits and 20 biscuits will be 5(0.5), 10(0.5) and 20(0.5) i.e., 2.5 cm, 5 cm and 10 cm respectively. Hence the ratio of their total surface area is 2π (2.5) [2.5 + 2.5] ; 2π (2.5) [5 + 2.5] ; 2π (2.5) [10 + 2.5] i.e. 5 : 7.5 : 12.5 i.e. 1 : 1.5 : 2.5 Choice (D) 111. The company buys printed packing sheets made of paper each having 100 × 66 cm2 area. At full capacity, the company makes 3.5 lakh biscuits per day. If it makes 10,000 packets each of the 5's, 10's and 20’s what is the minimum number of packing sheets required daily? (A) 417 (B) 419 (C) 420 (D) 418 Solution:

Total curved surface area of all the packets = 10,000 × (2π r) (h1 + h2 + h3) = 10,000 (2)

Hence total number of sheets required is Hence required Choice (A)

a

= 416.6

minimum

of

417

sheets

are

112. The cost of each biscuit is Rs.0.40, that of each packing sheet is Rs.168, and that of each end-cap is Rs.0.50. All other costs work out to Re.1 per packet, irrespective of the size of the packet. The maximum retail price of the 5's, 10's and 20's packets are Rs. 8, 14 and 23 respectively. The profits per packet, P5, P10 and P20 made by the company on the 5's, 10's and 20's packets respectively will be in the proportion: (A) 1 : 2 : 3 (B) 1 : 1.5 : 2.5 (C) 1 : 1.75 : 2.875 (D) 1 : 2 : 4 Solution: The following table gives the information of different cost involved in packing the biscuits Cost

Number of biscuits per pack 5

10

20

Biscuit

2

4

8

End-Cap

1

1

1

Packing sheet

1

2

4

Other

1

1

1

Total

5

8

14

Note: Calculation of packing sheet cost is given below.

Consider

the

pack

is

of

5

biscuits.

The

curved

surface

area

of

the

packet

=

Since the curved surface area of a pack of 10 biscuits is twice that of a pack of 5 biscuits its cost of packing sheet is Rs.2. Similarly the cost of packing sheet of a pack of 20 biscuits is Rs.4. ∴ P5 = 8 – 5 = 3 P10 = 14 – 8 = 6 P20 = 23 – 14 = 9 Hence P5 : P10 : P20 = 1: 2 : 3 Choice (A) 113. Based on a study of its sales over the last three years, the company decides to produce only 5000 packets of 20's. The production capacity this made available is used to produce additional 5's and 10's packets to meet the market demand. Let x1 and x2 respectively represent the additional numbers of 5's packets (in thousands), and 10's packets (in thousands). For every thousand of the additional 5's packets, the company has 15 distributors, and for every thousand of the additional 10's packets, it has 5 distributors. The company can utilize the services of a maximum of 75 distributors for these additional packets. Then the product-mix problem for producing the additional packet of 5's and 10's (in thousands) can be modeled using a Linear Programming formulation. Which of the following statements about this model is incorrect? (A) Maximize P5x1 + P10x2 can be the objective function (B) 15x1 + 5x2 ≤ 75 is a constraint (C) 5x1 + 10x2 = 100 is a constraint (D) x1 and x2 ≥ 0, and integers Solution: Given that the extra number of packets of 5 biscuits and 10 biscuits are x 1 and x2 (in thousands) respectively. From the data given in the previous questions, normally the company produces 10,000 packets each of 5’s , 10’s and 20’s. But now the company produces only 5000 packets of 20’s. So the remaining 1 lakh biscuits are packed into 5’s and 10’s. Hence 5x1 + 10x2 = 100. ∴ Choice (C) is correct. Also given that every thousand of the additional 5’s packets the company has 15 distributors and for every thousand of the 10’s packets it has 5 distributors. Since the company can utilize the services of a maximum of 75 distributors we have 15x1 + 15x2 ≤ 75. ∴ Choice (B) is also true. Clearly choice (A) is also true. Choice (D) is false because x1 and x2, since they are in thousands, need not be integers. Choice (D) 114. How profits? (A) (B) (C) (D)

many additional thousand packets of 5's and 10's should the company produce to maximize its x1 = 3; x2 = 7 x1 = 2; x2 = 10 x1 = 2; x2 = 9 solution is infeasible

Solution: Maximize Z= P5x1 + P10x2 subject to: 5x1 + 10x2 =100 → (1) 15x1 + 5x2 ≤ 75 → (2)

The feasible region is shown in the following graph.

The value of Z at N(0,10) is same as the value of Z at T (2, 9). Hence the value of Z will be maximum at any point on the segment NT. T (2, 9) is given in one choices. Choice (C)

of

the

115. The 5's, 10's and 20's biscuit packets are produced in lot sizes of 100 each. Three packets each of 5's, 10's and 20's are inspected at random. If even one biscuit is found broken in any of the three, the respective lot is rejected. the probability that a broken biscuit will be found in a 5's, 10's or 20's packet is estimated to be 0.10, 0.20 and 0.30 respectively. If a sample of three packets each of 5's, 10's and 20's is inspected, which probability distribution should we use to estimate the probability that all nine packets will be accepted? (A) Normal (B) Binominal (C) Poisson (D) Hypergeometric Solution: Consider 1 ‘Batch’ consisting of 1 lot of each size. Size

5

10

20

No of packets

100

100

100

Sample

3

3

3

P

0.1

0.2

0.3

Consider the 3 packets of 5’s in the sample. The possibilities and the corresponding probabilities are limited below No of packets containing broken biscuits

No of packets containing no broken biscuits

0

3

3

1

2

3

Probability C0(0.1)0(0.9)3 C1(0.1)1(0.9)2

2

1

3

3

0

3

We see distribution Choice (B)

that

we

C2(0.1)2(0.9)1 C1(0.1)3(0.9)0

should

116. What is the probability that all nine packets will be accepted? (A) 0.749 (B) 0.006 (C) 0.128

use

the

binomial

(D) 0.504

Solution: The probability that all the 3 packets are ok is (0.9)3. Similarly for the 10’s and 20’s the probability that all the 3 packets are ok is (0.8)3 and (0.7)3respectively. ∴ The probability that all the 9 packets are accepted is [(0.9) (0.8) (0.7)]3 ≈ 0.128 Choice (C) Directions: Questions 117 and 118 relate to the PROJECT INVESTMENT DECISIONS given below: A project requires an initial capital investment of Rs.2lakhs which yields a one time benefit at the end of the second year. 117. If the cost of capital is 10%, what should be the minimum yield that would justify an investment in this project? (A) Rs.2,42,000 (B) Rs.2,40,000 (C) Rs.2,20,000 (D) Rs.2,22,000 Solution: The total investment is 2 lakhs and there would be a profit only after two years. Since the cost of capital is 10% per year, by the end of second year the total cost of capital is 21% of 2 lakhs i.e. Rs.42,000. Hence the minimum profit that must be made from the business after two years is Rs.42,000. The business must yield a minimum of Rs.2,42,000 Choice (A) 118. Suppose the investment of Rs.2lakhs in the above project can be made in two equal instalments of Rs.1lakh in the beginning of the project and the other Rs.1lakh at the beginning of the second year. From the beginning of the third year, the project will generate revenues of Rs.1.5lakhs every year. Using the same cost of capital of 10%, what should be the minimum life of the project inclusive of the project implementation phase? (use the 'beginning of the year' convention) (A) 3 years (B) 5 years (C) 4 years (D) 6 years Solution: Let x years be the life of the project. The revenue from the project after x years is Rs.1.5 (x – 2). The investment of 2 lakhs is made in two equal instalments of Rs.1 lakh in the beginning of the project and the other 1 lakh at the beginning of second year. Since the cost of capital is 10% p.a. the worth of Rs.1 lakh invested

at the beginning of the project after ‘x’ years from the beginning is Rs.1 Rs.1 lakh invested at the beginning of second year

is Rs.1

Now, 1.5(x – 2) ≥ 1

and the worth of







⇒ From the given options x = 4 is the least value for which the above inequality will hold. Choice (C) Directions: Questions 119 and 120 relate to the COMPANY REVENUE MODELING problem given below: A company's analytics team has modeled its Total Revenue (TR) as TR = 3W1.2A3.6, where W is its wage payments and A is its advertising expenditure. 119. The approximate change in TR, if wages are increased by 5% will be (A) 3.15% (B) 5% (C) 3.18% (D) 6% Solution: Consider any function z of x and y of the form z = xmyn dz = mxm-1 yn dx

∴ In the given example, TR = zw1.2 A3.6

∴ Choice (D) 120. In addition to the said wage increase, if A is decreased by 2% the net change in TR will be (A) 3% (B) –1.2% (C) –0.04% (D) – 1.4% Solution: Given TR = 3W1.2 A3.6 From the previous question only it is clear that if W is increased by 5% TR will increase by 6%. Given that in addition to this A decreased by 2% If A3.6 will become (0.98)3.6 = (0.98)3.6 (A3.6) = (0.98)3.6 (0.98)0.6 A3.6 = (0.94) (0.98)0.6 (A3.6) = 0.94 × (0.98)0.5 × (0.98)0.1 × A3.6

= 0.94 × 0.99 × (0.98)0.1 × A3.6 Let the new value of TR be TR1 ∴ TR1 = 3 [1.06 × 0.94 × 0.99 × (0.98) 0.1] w1.2 A 3.6 TR1 = 3 [ 0.9864 × (0.98)0.1] w1.2A3.6 ⇒ TR1 < (0.9864)TR ∴ TR1 is less than TR by more than 1.36% Alternate Solution: TR = ZW1.2 A 3.6 Consider any function z of x and y of the form z = xmyn If x increases by 4x, xm increases approximately by m ∆ x If y decreases by ∆ y, yn decreases approximately by n ∆ y. ∴ z + ∆ z = (x + m ∆ x) (y − n ∆ y) = xy + my ∆ x − nx ∆ y − mn ∆ x ∆ y or ∆ z = my ∆ x − nx ∆ y − mn ∆ x ∆ y

or

In the given example m = 1.2, n = 3.6



= 1.2 (0.05) − 3.6 (0.02) − (1.2) (3.6) (0.05) (0.02)

Multiplying this by 100, we get the change in z in percentage points as 6 − 7.2 − 0.432 = 1.632 The closest value is −1.4% Choice (D)

SECTION 4: DATA INTERPRETATION (121 – 150) Directions: Questions 121 to 126 are based on the information given below: The Market share of five types of products manufactured by a company in the year 2006-07 is plotted against their Business Strength and Industry Attractiveness Indices (read at the centre of each bubble) in the graph below. Both indices range from 0.0 (minimum) to 1.0 (maximum). The size of each bubble is representative of the Market Share of each product (printed beside the respective bubble) as a percentage of the Total Market Size in the year 2006-07.

The Total Market Size for various products in 2006-07 and their Profit % are given below: Product

Total Market Size in million Rs.

Profit %

Bearings

2200

40

Valves

1100

60

Pipes

3000

30

Belts

2500

30

Gears

5000

20

The Total Market Size of a product is estimated to increase yearly at the same rate as its Industry Attractiveness Index. that is, an Industry Attractiveness Index of 0.6 would imply that total Market Size will be 60% more than that of the previous year. A business Strength Index of 0.4 would imply that the Market Share of the company will increase by 40% of the total Market Share held by other companies during the previous year. 121. The combined profit (in million Rs.) of the three most profitable products in the year 2006-07 is approximately (A) 1092 (B) 1102 (C) 1233 (D) 1328 Solution: The given values can be tabulated as follows: Total market (in million Rs.)

Company Share (in million Rs.)

Profit %

Company Profit (in million Rs.)

Bearings

2200

880

40

352

Valves

1100

220

60

132

Pipes

3000

1500

30

450

Belts

2500

750

30

225

Gears

5000

1500

20

300

Product

The combined profit of the three most profitable products = 352 + 450 +300 = 1102 Choice (B) 122. Which is the most profitable product in the year 2006-07? (A) Bearings (B) Pipes (C) Belts Solution: The pipes. Choice (B)

most

profitable

123. Which product would be least profitable in the year 2007-08? (A) Bearings (B) Valves (C) Pipes Solution:

(D) Gears product

(D) Belts

is

The values for 2007-08 can be tabulated as follows. Product

Total market (million Rs.)

Company share (million Rs.)

Profit (%)

Company profit (million Rs.)

Bearing

3960

40% + 60% of 60% = 76% = 3010

40

1204

Valves

1760

20% + 40% of 80% = 52% = 915

60

549

Pipes

5700

50% + 90% of 50% = 95% = 5415

30

1624.5

Belts

3500

30% + 60% of 70% = 72% = 2520

30

756

Gears

8500

30% + 60% of 70%= 72% = 6120

20

1224

Values profitable

are

the

least

Choice (B) 124. The company decided to discontinue the production of Valves and Belts. The Total Profit (in million Rs.) of the company from the remaining three products in 2007-08 would be approximately (A) 3164 (B) 3454 (C) 3844 (D) 4052 Solution: Total profit from bearings, pipes and gears is = 1204 + 1224 µ ≃ 4052 Choice (D)

1624.5

+

125. The Total Market Size(in million Rs.) for all products put together in the year 2007-08 is approximately (A) 17555 (B) 19230 (C) 21350 (D) 23420 Solution: Total market 23420

size

=

3960

+

1760

+

5700

+

3500

+

8500 = Choice (D)

126. The Total Market Size for gears is assumed to continue to grow every year by 70%. What will be the approximate Total Market Size for gears (in million Rs.) in the year 2008-09? (A) 14450 (B) 15550 (C) 16550 (D) 17450 Solution: Market size for gears in 2008 – 09 = 8500 × 1.7 450 Choice (A)

=

14,

Directions: Questions 127 to 132 are based on the information given below: A company manufacturing and selling vacuum cleaners started operations with cash in hand of Rs. 5 million at the beginning of 2002-03. The table below gives the production, sales, price and costs of the company over the next five years. Year

2002-03

2003-04

2004-05

2005-06

2006-07

Production (units)

14000

18000

20000

17000

15000

Sales (units)

12000

17000

16000

19000

19000

Price (Rs./unit)

10000

11000

11000

11000

12000

Fixed cost (million Rs.)

18

30

30

40

40

Total Variable cost (million Rs.)

84

122

141

161

172

127. At the end of which year was the quantity of cumulative unsold stock the largest? (A) 2002-03 (B) 2003-04 (C) 2004-05 (D) None of the above Solution: The values for the given years can be tabulated as follows. Year

2002-03

2003-04

2004-05

2005-06

2006-07

14000

18000

20000

17000

15000

10.2

15.2

17.1

20.1

21.2

12

18.7

17.6

20.9

22.8

Unsold Stock (Cumulative units)

2000

3000

7000

5000

1000

Cash in hand (Cumulative) (Cr.)

1.8 +0.5 = 2.3

5.8

6.3

7.1

8.7

Production (Units) Total Cost (Fixed + Variable) (Cr.) Sales in Rupees (Units × Price)(Cr.)

The

cumulative

unsold

stock

was

the

highest

at

the

05.

end of Choice (C)

2004-

128. Assuming all cash transactions, the cumulative cash in hand with the company (in million Rs.) at the end of 2006-07 is (A) 16 (B) 20 (C) 82 (D) 87 Solution: The values for the given years can be tabulated as follows. Year

2002-03

2003-04

2004-05

2005-06

2006-07

14000

18000

20000

17000

15000

Total Cost (Fixed + Variable) (Cr.)

10.2

15.2

17.1

20.1

21.2

Sales in Rupees (Units × Price) (Cr.)

12

18.7

17.6

20.9

22.8

Unsold Stock (Cumulative units)

2000

3000

7000

5000

1000

Cash in hand (Cumulative) (Cr.)

1.8 +0.5 = 2.3

5.8

6.3

7.1

8.7

Production (Units)

The cumulative millions.

cash

at

the

end

of

2006-07

was

the

highest and Choice (D)

is

129. The percentage increase in cumulative cash in hand over the preceding year was the highest in (A) 2002-03 (B) 2003-04 (C) 2004-05 (D) 2005-06 Solution: The values for the given years can be tabulated as follows.

Rs.87

Year

2002-03

2003-04

2004-05

2005-06

2006-07

14000

18000

20000

17000

15000

Total Cost (Fixed + Variable) (Cr.)

10.2

15.2

17.1

20.1

21.2

Sales in Rupees (Units × Price) (Cr.)

12

18.7

17.6

20.9

22.8

Unsold Stock (Cumulative units)

2000

3000

7000

5000

1000

Cash in hand (Cumulative) (Cr.)

1.8 +0.5 = 2.3

5.8

6.3

7.1

8.7

Production (Units)

In 2003-04, the cumulative cash in hand is more than doubled when compared to the previous year, which is the highest percentage. Choice (B) 130. The increase in the difference between income and costs in a year over the preceding year was the largest in (A) 2003-04 (B) 2004-05 (C) 2005-06 (D) 2006-07 Solution: The values for the given years can be tabulated as follows. Year

2002-03

2003-04

2004-05

2005-06

2006-07

14000

18000

20000

17000

15000

Total Cost (Fixed + Variable) (Cr.)

10.2

15.2

17.1

20.1

21.2

Sales in Rupees (Units × Price) (Cr.)

12

18.7

17.6

20.9

22.8

Unsold Stock (Cumulative units)

2000

3000

7000

5000

1000

Cash in hand (Cumulative) (Cr.)

1.8 +0.5 = 2.3

5.8

6.3

7.1

8.7

Production (Units)

The increase in difference between income and cost in an year over the preceding year was the highest in 2003-04 (3.5 − 1.8 = 1.7 Cr.) Choice (A) 131. If the company was able to sell all the units that it produced during a year in the same year, the cash in hand (in million Rs.) at the end of 2006-07 would be (A) 87 (B) 92 (C) 102 (D) 117 Solution: The values for the given years can be tabulated as follows. Year

2002-03

2003-04

2004-05

2005-06

2006-07

14000

18000

20000

17000

15000

Total Cost (Fixed + Variable) (Cr.)

10.2

15.2

17.1

20.1

21.2

Sales in Rupees (Units × Price)

12

18.7

17.6

20.9

22.8

Production (Units)

(Cr.) Unsold Stock (Cumulative units)

2000

3000

7000

5000

1000

Cash in hand (Cumulative) (Cr.)

1.8 +0.5 = 2.3

5.8

6.3

7.1

8.7

Total sales value = 14 + 19.8 + 22 +18.7 + 18 = 92.5 Total cost = 10.2 +15.2 +17.1 + 20.1 + 21.2 = 83.8 Required difference = 8.7 + 0.5 millions. hoice (B)

=

9.2

Cr.

Or

92 C

132. Assuming that all variable costs incurred in a year are attributable to the units produced in the year, the highest variable cost per unit was incurred in the year (A) 2002-03 (B) 2004-05 (C) 2006-07 (D) None of the above Solution: The values for the given years can be tabulated as follows. Year

2002-03

2003-04

2004-05

2005-06

2006-07

14000

18000

20000

17000

15000

Total Cost (Fixed + Variable) (Cr.)

10.2

15.2

17.1

20.1

21.2

Sales in Rupees (Units × Price) (Cr.)

12

18.7

17.6

20.9

22.8

Unsold Stock (Cumulative units)

2000

3000

7000

5000

1000

Cash in hand (Cumulative) (Cr.)

1.8 +0.5 = 2.3

5.8

6.3

7.1

8.7

Production (Units)

As the production in 2006-07 is one of the lowest and the variable costs that year is the highest, the variable cost per unit was the highest in the year 2006-07. Choice (C) Directions: Questions 133 to 138 are based on the information given below: The Venn-diagram given below shows the estimated readership of three daily newspapers (X, Y and Z) in a city.

The total readership and advertising cost for each of these papers is as follows: Newspaper

Readership (lakhs)

Advertising cost (Rs.per sq.cm)

X

8.7

6000

Y

9.1

6500

Z

5.6

5000

The total population of the city is estimated to be 14 million. The common readership (in lakhs) is indicated in the above Venn-diagram) 133. The number of people (in lakhs) who read at least one newspaper is (A) 4.7 (B) 11.9 (C) 17.4 (D) 23.4 Solution: The complete venn diagram is as follows:

The number of people who read at least one news paper = 4.7 + 2.5 + 0.5 + 1.0 + 2.6 +1.5 + 4.6 = 17.4 Choice (C) 134. The number of people (in lakhs) who read only one news paper is (A) 4.7 (B) 11.9 (C) 17.4 (D) 23.4 Solution: The complete venn diagram is as follows:

11.9

The number of people who read only one newspaper = 4.7 + 4.6

+

Choice (B) 135. The approximate percentage of population reading at least two newspapers is (A) 2.9 (B) 3.5 (C) 3.9 (D) None of the above Solution: The complete venn diagram is as follows:

2.6

=

Approximate

percentage

of

population

=

reading

at

least

two

news

papers

Choice (C)

136. The combination of any two newspapers that gives the minimum advertising cost (in Rs.per sq.cm.) per 1000 readers is (A) X and Y (B) Y and Z (C) X and Z (D) None of the above Solution: The complete venn diagram is as follows:

The number of people reading the combination of papers and the advertising costs for them are:

Combination X and Y Y and Z X and Z The Y.

ratio

of

cost

per

Readership 14.8 12.7 12.8

readers

is

the

Cost 12,500 11,500 11,000

lowest

for

the

combination Choice (A)

X

and

137. The ratio of readers reading only one newspaper to those reading only two newspapers is (A) 2.38 : 1 (B) 3.65 : 1 (C) 4.57 : 1 (D) None of the above Solution: The complete venn diagram is as follows:

Required 1

ratio

=

11.9

:

5

=

2.38

:

Choice (A)

138. The minimum expenditure(in Rs. per sq. cm.) on advertising required to reach at least 12 lakh readers is (A) 11000 (B) 11500 (C) 12500 (D) None of the above Solution: The complete venn diagram is as follows:

A combination of X and Z would have a reach of 12.8 lakh readers and the expenditure would be Rs.11,000, which is the minimum expenditure reach over 12 lakh readers. Choice (A) Directions: Questions 139 to 144 are based on the information given below: Data on edible oil production and consumption for the year 2006-07 across six countries is tabulated below:

Country

Annual edible oil production (billion liters)

Approximate population (million)

Annual edible oil consumption per capita (liters)

Projected annual population growth rate %

Projected annual Projected annual edib population growth consumption growth rate % per capita %

I

12

1000

12

10

2

5

J

7

500

13

5

5

6

K

5

300

11

5

4

7

L

10

1200

10

10

1

1

M

9

700

14

5

4

6

139. Assuming none of the above five countries export edible oil, which country would need to import the maximum quantity of edible oil in 2007-08? (A) I (B) J (C) K (D) None of the above Solution: As 1000 million = 1 billion, the annual edible oil production and edible oil consumption in different countries in 2007-08 would be

Country

Production (in billion)

Population

Per capita Consumption

Total Consumption (in billion)

I

13.2

1.02

12.60

12.852

J K L M ∴ Country quantity. ce (D)

7.35 5.25 11 9.45 M

0.525 0.312 1.212 0.728 would

have

13.78 11.77 10.10 14.84 to

7.2345 3.672 12.24 10.80 import

the

maximum Choi

140. Assuming that a country imports edible oil only to meet the gap between production and consumption, if any, which country is in a position to export the maximum quantity of edible oil in 2006-07? (A) I (B) J (C) K (D) None of the above Solution: From the values in tables, it can easily be concluded that country K can export the maximum quantity in 2006-07. Choice (C) 141. Assuming none of the above five countries exported or imported edible oil, which country that had an export capacity in 2006-07 would need to import next year? (A) I (B) J (C) L (D) None of the above Solution: From the tables given only countries J K and M have an export capacity in the year 2006-07. From the table for Q.133 we can see that in the year 2007-08 only M need to import oil. Choice (D) 142. What is the approximate average per capita consumption of edible oil (in liters) for all the five countries put together in the year 2008-09? (A) 11.52 (B) 12.33 (C) 13.27 (D) 14.31 Solution: From the choices given, the question asked has to be ‘what is the approximate average of per capita consumption … The approximate values of per capita consumption for the different countries in 2008-09 are I - 13.23 J - 14.60 K - 12.59 L - 10.20 M - 15.73 Total- 66.35

Average

= Choice (C)

143. Which country would have the highest annual consumption of edible oil in liters per capita in the year 2008-09? (A) I (B) K (C) L (D) M Solution: As the annual per capita consumption of country M is the highest in the year 2006-07 and it has the highest growth rate in per capita consumption among all the countries with high values of per capita consumption, country M would have the highest value of per capita consumption in the year 2008 – 09. Choice (D) 144. The net import requirement of edible oil (in million liters) for all the five countries put together in the year 2007-08 is closest to (A) 450 (B)550 (C) 650 (D) 750

Solution: From the table given for Q.133, the net import requirement would be (− 0.35) + (− 0.12) + (− 1.58) + 1.24 + 1.35 = 0.54 billions ≅ 550 litres. Choice (B) Directions: Questions 145 to 150 are based on the information given below:

million

A departmental store reported the following sales data (in Million Rs.) for a particular week:

Day

Section Personal Men's Aarel products 0.61 0.42

Ladies' Apparel 0.72

Grocery

Confectionary

Appliances

Monday

0.18

0.12

Tuesday

0.16

0.14

0.72

0.38

0.56

0.68

Wednesday

0.22

0.21

0.86

0.46

0.68

0.76

Thursday

0.28

0.16

0.68

0.54

0.58

0.82

Friday

0.29

0.32

0.88

0.61

0.42

0.54

Saturday

0.33

0.42

1.10

0.92

0.88

0.96

Sunday

0.38

0.58

1.22

1.42

1.46

1.42

0.86

The Average Value of a Transaction (AVT) in Rs. and the Time Taken per Transaction (TpT) in seconds is given in the figure below:

time.

Note: The store assigns one salesperson in a section in a section for every 400 minutes of transaction

145. The total sales of all the sections put together is the same on (A) Monday and Tuesday (B) Tuesday and Wednesday (C) Wednesday and Thursday (D) Thursday and Friday Solution: The total sales of all the sections on different days are Monday : 0.18 + 0.12 + 0.61 + 0.42 + 0.72 + 0.86 = 2.91 Tuesday : 0.16 + 0.14 + 0.72 + 0.38 + 0.56 +0.68 = 2.64 Wednesday : 0.22+0.21+0.86+0.46+0.68+0.76 = 3.19 Thursday : 0.28 + 0.16 + 0.68 +0.54 +0.58 + 0.82 = 3.06 Friday : 0.29 + 0.32 + 0.88 + 0.61 + 0.42 + 0.54 = 3.06 It is the same Friday. Choice (D)

on

Thursday

and

146. The ratio of sales on the week-end (Saturday and Sunday) to total sales is nearly (A) 1 : 2.34 (B) 1 : 3.43 (C) 1 : 4.48 (D) 1 : 5.51 Solution: Sales on Saturday and Sunday = 4.61 +6.48 = 11.09 Total sales = 2.91 + 2.64 + 3.19 +3.06 +3.06 +11.09 = 25.95

Required 2.34 Choice (A)

ratio

=

=

1

:

147. The section reporting the highest transaction time on Sunday is (A) Grocery (B) Confectionary (C) Personal Products (D) None of the above Solution: From the total sales and the average value of transaction, we can find the number of transactions which is the highest for confectionary, closely followed by Grocery. But as average time per transaction for a grocery is double that of confectionary, on the total transaction time would be the highest for Grocery. Even though ladies apparel and appliances have more average time per transaction, when compared to groceries, the number of transactions is much lower and so total transaction time would be lower. Choice (A) 148. The section with the highest number of transactions on Sunday is

(A) Grocery (B) Confectionary (C) Personal Products (D) None of the above Solution: The number of transaction, for grocery and confectionary is much more than others.

Among .

these

grocery

is

and

confectionary

is and Choice (B)

>

149. The section requiring the maximum number of salespersons on Saturday is (A) Grocery (B) Confectionary (C) Personal Products (D) None of the above Solution: As one sales person is required for every 400 minutes of transaction time, the maximum sales persons would be required in the section in which the transaction time is the maximum, i.e., Grocery. Choice (A) 150. The ratio of salespersons required in the Appliances section on Monday to the salespersons required in the same section on Sunday is approximately (A) 1 : 0.50 (B) 1 : 1.45 (C) 1 : 1.65 (D) 1 : 1.95 Solution: As one sales person is required for every 400 minutes of transaction time, and as average value of transaction for appliances on Monday and Sunday is the same, the ratio of total sales on Monday and Sunday would give the ratio of the number of sales persons, required, i.e. 86 : 1.42 = 1:1.65. Choice (C)

Related Documents


More Documents from ""